Nothing Special   »   [go: up one dir, main page]

Eckonomy - Pyq - Workbook 1

Download as pdf or txt
Download as pdf or txt
You are on page 1of 58

PRELIMS PRACTICE

WORKBOOK
ECONOMY

CONTENTS
 ECONOMY PREVIOUS YEAR QUESTIONS ............................................................ 01-22
1. Basic concepts ................................................................................................................... 1
2. Economic Measurements ................................................................................................. 3
3. Money And Banking .......................................................................................................... 6
4. Economic Sectors............................................................................................................. 12
5. Socio-Economic Sectors .................................................................................................. 19

 ECONOMY ANSWERS............................................................................................ 23-50


1. Basic Concepts ................................................................................................................. 23
2. Economic Measurements ............................................................................................... 25
3. Money And Banking ........................................................................................................ 29
4. Economic Sectors............................................................................................................. 37
5. Socio-Economic Sectors .................................................................................................. 46
ECONOMY
PREVIOUS YEAR QUESTIONS

BASIC CONCEPTS

1. Despite being a high saving economy, capital (c) There is capital formation in X
formation may not result in significant increase in
(d) The volume of trade grows in the world economy
output due to
(a) Weak administrative machinery 6. Consider the following specific stages of
(b) Illiteracy demographic transition associated with economic
development:
(c) High population density
1. Low birth rate with a low death rate
(d) High capital output ratio 2. High birth rate with a high death rate
2. If a commodity is provided free to the public by the 3. High birth rate with a low death rate
government, then: Select the correct answer using the codes given
(a) The opportunity cost is zero. below:

(b) The opportunity cost is ignored. (a) 1, 2 and 3 only (c) 2, 3 and 1 only

(c) The opportunity cost is transferred from the (b) 3, 2 and 1 only (d) 3, 2 and 1 only
consumers of the product to the taxpaying
public. 7. Microfinance is the provision of financial services
to people of low-income groups. This includes both
(d) The opportunity cost is transferred from the the consumers and the self-employed. The service/
consumers of the product to the government. services rendered under micro-finance is/are:
1. Credit facilities
3. What does venture capital mean?
2. Savings facilities
(a) A short-term capital provided to industries 3. Insurance facilities
(b) A long-term start-up capital provided to new 4. Fund Transfer facilities
entrepreneurs
Select the correct answer using the codes given below
(c) Funds provided to industries at times of incurring the lists:
losses
(a) 1 only (c) 2 and 3 only
(d) Funds provided for replacement and renovation
of industries (b) 1 and 4 only (d) 1, 2, 3 and 4

4. If the interest rate is decreased in an economy, it 8. As a result of their annual survey, the National
will Geographic Society and an international polling
firm GlobeScan gave India top rank in Greendex
(a) Decrease the consumption expenditure in the 2009 score. What is this score?
economy
(a) It is a measure of efforts made by different
(b) Increase the tax collection of the Government countries in adopting technologies for reducing
(c) Increase the investment expenditure in the the carbon footprint
economy (b) It is a measure of environmentally sustainable
(d) Increase the total savings in the economy consumer behavior in different countries
(c) It is an assessment of programs/schemes
5. Economic growth in country X will necessarily undertaken by different countries for improving
have to occur if the conservation of natural resources
(a) There is technical progress in the world economy (d) It is an index showing the volume of carbon
(b) There is population growth in X credits sold by different countries

1
ECONOMY WORKBOOK www.iasscore.in

9. If another global financial crisis happens in the 3. The higher a borrowing firm’s level of Interest
near future, which of the following actions/policies Coverage Ratio, the worse is its ability to service
are most likely to give some immunity to India? its debt.
1. Not depending on short-term foreign Select the correct answer using the code given below:
borrowings (a) 1 and 2 only (c) 1 and 3 only
2. Opening up to more foreign banks (b) 2 only (d) 1, 2 and 3
3. Maintaining full capital account convertibility
14. Consider the following statements:
Select the correct answer using the code given below:
1. The weightage of food in Consumer Price Index
(a) 1 only (c) 2 only (CPI) is higher than that in Wholesale Price Index
(b) 1 and 2 only (d) 1, 2 and 3 (WPI)
2. The WPI does not capture changes in the prices
10. If you withdraw Rs. 1,00,000 in cash from your of services, which CPI does.
Demand Deposit Account at your bank, the 3. Reserve Bank of India has now adopted WPI as
immediate effect on aggregate money supply in the its key measure of inflation and to decide on
economy will be changing the key policy rates.
(a) To reduce it by Rs. 1,00,000 Which of the statements given above is/are correct?
(b) To increase it by Rs. 1,00,000 (a) 1 and 2 only (c) 3 only
(c) To increase it by more than Rs. 1,00,000 (b) 2 only (d) 1, 2 and 3

(d) To leave it unchanged 15. “Rapid Financing Instrument” and “Rapid Credit
Facility” are related to the provisions of lending by
11. With reference to Foreign Direct Investment in which one of the following?
India, which one of the following is considered its
(a) Asian Development Bank
major characteristic?
(b) International Monetary Fund
(a) It is the investment through capital instruments
essentially in a listed company. (c) United Nations Environment Programme Finance
Initiative
(b) It is a largely non-debt creating capital flow
(d) World Bank
(c) It is the investment which involves debt-
servicing. 16. With reference to the Indian economy, consider
the following statements:
(d) It is the investment made by foreign institutional
investors in the Government securities. 1. An increase in Nominal Effective Exchange Rate
(NEER) indicates the appreciation of rupee.
12. With reference to the Indian economy after the 2. An increase in the Real Effective Exchange
1991 economy liberalization, consider the following Rate (REER) indicates an improvement in trade
statements competitiveness.
1. Worker productivity (`per worker at 2004- 3. An increasing trend in domestic inflation relative
05 prices) increased in urban areas while it to inflation in other countries is likely to cause an
decreased in rural areas. increasing divergence between NEER and REER.

2. The percentage share of rural areas in the Which of the above statements are correct?
workforce steadily increased. (a) 1 and 2 only (c) 1 and 3 only
3. In rural areas, the growth in non-farm economy (b) 2 and 3 only (d) 1, 2 and 3
increased
4. The growth rate in rural employment decreased. 17. With reference to the Indian economy, consider
the following statements:
Which of the statements given above is/are correct? 1. If the inflation is too high, Reserve Bank of India
(a) 1 and 2 only (c) 3 only (RBI) is likely to buy government securities.
(b) 3 and 4 only (d) 1 2 and 4 only 2. If the rupee is rapidly depreciating, RBI is likely
to sell dollars in the market.
13. What is the importance of the term “Interest 3. If interest rates in the USA or European Union
Coverage Ratio” of a firm in India? were to fall, that is likely to induce RBI to buy
dollars.
1. Its helps in understanding the present risk of a
firm that a bank is going to give loan to. Which of the statements given above are correct?
2. Its helps in evaluating the emerging risk of a firm (a) 1 and 2 only (c) 1 and 3 only
that a bank is going to give loan to. (b) 2 and 3 only (d) 1, 2 and 3

2
www.iasscore.in ECONOMY WORKBOOK

ECONOMIC MEASUREMENTS

1. Consider the following: (c) a type of systemic risk that arises where perfect
1. Demographic performance hedging is not possible.
2. Forest and ecology (d) a numeric value that measures the fluctuations of
3. Governance reforms a stock to changes in the overall stock market.
4. Stable government 5. Consider the following statements:
5. Tax and fiscal efforts
Human capital formation as a concept is better
For the horizontal tax devolution, the Fifteenth explained in terms of a process which enables.
Finance Commission used how many of the above 1. Individuals of a country to accumulate more
as criteria other than population area and income capital.
distance?
2. Increasing the knowledge, skill levels and
(a) Only two (c) Only four capacities of the people of the country.
(b) Only three (d) All five 3. Accumulation of tangible wealth.
4. Accumulation of intangible wealth.
2. Which one of the following activities of the
Reserve Bank of India is considered to be part of Which of the statements given above is/are correct?
‘sterilization’? (a) 1 and 2 only (c) 2 and 4 only
(a) Conducting ‘Open Market Operations’ (b) 2 only (d) 1, 3 and 4 only
(b) Oversight of settlement and payment systems
6. Increase in absolute and per capita real GNP do not
(c) Debt and cash management for the Central and
connote a higher level of economic development,
State Governments
if:
(d) Regulating the functions of Non-banking Financial
(a) Industrial output fails to keep pace with agriculture
Institutions
output.
3. Consider the following statements: (b) Agriculture output fails to keep pace with industrial
output.
Statement-I:
(c) Poverty and unemployment increase.
Interest income from the deposits in Infrastructure
Investment Trusts (InvITs) distributed to their (d) Imports grow faster than exports.
investors is exempted from tax, but the dividend is
taxable. 7. As per the NSSO 70th Round “Situation Assessment
Survey of Agricultural Households”, consider the
Statement-II:
following statements:
InvITs are recognized as borrowers under the 1. Rajasthan has the highest percentage share
“Securitization and Reconstruction of Financial Assets of agricultural households among its rural
and Enforcement of Security Interest-Act, 2002: households.
Which one of the following is correct in respect of the 2. Out of the total agricultural households in the
above statements? country, a little over 60 percent belong to OBCs.
(a) Both Statement-I and Statement-II are correct 3. In Kerala, a little over 60 percent of agricultural
and Statement-II is the correct explanation for households reported to have received maximum
Statement-I income from sources other than agricultural
(b) Both Statement-I and Statement-II are correct and activities.
Statement-II is not the correct explanation for Which of the statements given above is/are correct?
Statement-I
(a) 2 and 3 only (c) 1 and 3 only
(c) Statement-I is correct but Statement-II is incorrect
(b) 2 only (d) 1, 2 and 3
(d) Statement-I is incorrect but Statement-II is correct
8. With reference to the Indian economy, consider
4. In the context of finance, the term ‘beta’ refers to the following statements:
(a) the process of simultaneous buying and selling of 1. The rate of growth of Real Gross Domestic
an asset from different platforms. Product has steadily increased in the last decade.
(b) an investment strategy of a portfolio manager to 2. The Gross Domestic Product at market prices (in
balance risk versus reward. rupees) has steadily increased in the last decade.

3
ECONOMY WORKBOOK www.iasscore.in

Which of the statements given above is/are correct? 14. Under which of the following circumstances may
‘capital gains‘ arise?
(a) 1 only (c) Both 1 and 2
1. When there is an increase in the sales of a
(b) 2 only (d) Neither 1 nor 2 product
2. When there is a natural increase in the value of
9. With reference to inflation in India, which of the the property owned.
following statements is correct?
3. When you purchase a painting and there is
(a) Controlling the inflation in India is the a growth in its value due to an increase in its
responsibility of the Government of India only popularity.
(b) The Reserve Bank of India has no role in Select the correct answer using the codes given below:
controlling the inflation (a) 1 only (c) 2 only
(c) Decreased money circulation helps in controlling
the inflation (b) 2 and 3 only (d) 1, 2 and 3

(d) Increased money circulation helps in controlling 15. India has experienced persistent and high food
the inflation inflation in the recent past. What could be the
reasons?
10. A rise in the general level of prices may be caused 1. Due to a gradual switchover to the cultivation of
by commercial crops, the area under the cultivation
1. An increase in the money supply of food grains has steadily decreased in the last
2. A decrease in the aggregate level of output five years by about 30%.

3. An increase in the effective demand 2. As a consequence of increasing incomes, the


consumption patterns of the people have
Select the correct answer using the code given below: undergone a significant change.
(a) 1 only (c) 2 and 3 only 3. The food supply chain has structural constraints.
(b) 1 and 2 only (d) 1, 2 and 3 Which of the statements given above are correct?

11. Which one of the following is likely to be the most (a) 1 and 2 only (c) 1 and 3 only
inflationary in its effect? (b) 2 and 3 only (d) 1, 2 and 3
(a) Repayment of public debt
16. A rapid increase in the rate of inflation is
(b) Borrowing from the public to finance a budget sometimes attributed to the “base effect”. What is
deficit the “base effect”?
(c) Borrowing from banks to finance a budget deficit (a) It is the impact of drastic deficiency in supply due
to failure of crops
(d) Creating new money to finance a budget deficit
(b) It is the impact of the surge in demand due to
12. The national income of a country for a given period rapid economic growth
is equal to the (c) It is the impact of the price levels of previous year
(a) The total value of goods and services produced by on the calculation of inflation rate
the nationals (d) None of the statements (a), (b) and (c) ‘given above
(b) Sum of total consumption and investment is correct in this context
expenditure
17. In the context of Indian economy, consider the
(c) Sum of personal income of all individuals
following statements:
(d) The money value of final goods and services 1. The growth rate of GDP has steadily increased in
produced the last five years.
2. The growth rate in per capita income has steadily
13. Consider the following statements:
increased in the last five years.
1. Inflation benefits the debtors.
Which of the statements given above is/are correct?
2. Inflation benefits the bond-holders.
(a) 1 only (c) Both 1 and 2
Which of the statements given above is/are correct?
(b) 2 only (d) Neither 1 nor 2
(a) 1 only
(b) 2 only 18. Economic growth is usually coupled with
(c) Both 1 and 2 (a) Deflation (c) Stagflation
(d) Neither 1 nor 2 (b) Inflation (d) Hyperinflation

4
www.iasscore.in ECONOMY WORKBOOK

19. Which one of the following statements is an declined in the last 10 years
appropriate description of deflation?
Which of the statements given above is/are correct?
(a) It is a sudden fall in the value of a currency
against other currencies (a) 1 only (c) Both 1 and 2

(b) It is a persistent recession in both the financial (b) 2 only (d) Neither 1 nor 2
and real sectors of the economy
24. Which among the following steps is most likely to
(c) It is a persistent fall in the general price level of be taken at the time of an economic recession?
goods and services
(a) Cut in tax rates accompanied by increase in
(d) It is a fall in the rate of inflation over a period of interest rate
time
(b) Increase in expenditure on public projects
20. In the context of Indian economy, consider the (c) Increase in tax rates accompanied by reduction
following pairs: of interest rate
Term Most appropriate description (d) Reduction of expenditure on public projects
1. Melt down Fall in stock prices
25. Consider the following statements: Other things
2. Recession Fall in growth rate remaining unchanged, market demand for a good
3. Slow down Fall in GDP might increase if
1. Price of its substitute increases
Which of the pairs given above is/are correctly
matched? 2. Price of its complement increases

(a) 1 only (c) 1 and 3 only 3. The good is an inferior good and income of the
consumers increases
(b) 2 and 3 only (d) 1, 2 and 3
4. Its price falls
21. Consider the following actions by the Government: Which of the above statements are correct?
1. Cutting the tax rates (a) 1 and 4 only (c) 1, 3 and 4
2. Increasing government spending (b) 2, 3 and 4 (d) 1, 2 and 3
3. Abolishing the subsidies
26. With reference to Indian economy, demand-pull
In the context of economic recession, which of the
inflation can be caused/increased by which of the
above actions can be considered a part of the “Fiscal
following?
stimulus” package?
1. Expansionary policies
(a) 1 and 2 only (c) 1 and 3 only
2. Fiscal stimulus
(b) 2 only (d) 1, 2 and 3 3. Inflation-indexing wages
4. Higher purchasing power
22. With reference to India, consider the following
statements: 5. Rising interest rates
1. The Wholesale Price Index (WPI) in India is Select the correct answer using the code given below.
available on a monthly basis only (a) 1, 2 and 4 only (c) 1, 2, 3 and 5 only
2. As compared to Consumer Price Index for
(b) 3, 4 and 5 only (d) 1, 2, 3, 4 and 5
Industrial Workers (CPI(IW)), the WPI gives less
weight to food articles 27. Consider the following statements:
Which of the statements given above is/are correct? 1. In India, credit rating agencies are regulated by
(a) 1 only (c) Both 1 and 2 Reserve Bank of India.
(b) 2 only (d) Neither 1 nor 2 2. The rating agency popularly known as ICRA is a
public limited company.
23. With reference to the Indian economy, consider 3. Brickwork Ratings is an Indian credit rating
the following statements: agency.
1. The Gross Domestic Product (GDP) has increased Which of the statements given above are correct?
by four times in the last 10 years (a) 1 and 2 only (c) 1 and 3 only
2. The percentage share of Public Sector in GDP has (b) 2 and 3 only (d) 1, 2 and 3

5
ECONOMY WORKBOOK www.iasscore.in

MONEY AND BANKING

1. Consider the following statements 5. Which of the following is not included in the assets
of a commercial bank in India?
Statement-I:
(a) Advances
In the post-pandemic recent past, many Central Banks
worldwide had carried out interest rate hikes. (b) Deposits
Statement-II: (c) Investments
Central Banks generally assume that they have the (d) Money at call and short notice
ability to counteract the rising consumer prices via
monetary policy means. 6. Which one of the following is not the most likely
measure the Government/RBI takes to stop the
Which one of the following is correct in respect of the slide of Indian rupee?
above statements?
(a) Curbing imports of non-essential goods and
(a) Both Statement-I and Statement-II are correct promoting exports
and Statement-II is the correct explanation for
(b) Encouraging Indian borrowers to issue rupee
Statement-I
denominated Masala Bonds
(b) Both Statement-I and Statement-II are correct
(c) Easing conditions relating to external commercial
and Statement-II is not the correct explanation borrowing
for Statement-I
(d) Following an expansionary monetary policy
(c) Statement-I is correct but Statement II is incorrect
(d) Statement-I is incorrect but Statement –II is 7. The money multiplier in an economy increases
correct with which one of the following?
(a) Increase in the cash reserve ratio
2. With reference to Central Bank digital currencies,
(b) Increase in the banking habit of the population
consider the following statements:
1. It is possible to make payments in a digital (c) Increase in the statutory liquidity ratio
currency without using US dollar or SWIFT (d) Increase in the population of the country
system.
2. A digital currency can be distributed with a 8. Consider the following items:
condition programmed into it such as a time- 1. Cereal grains hulled
frame for spending it. 2. Chicken eggs cooked
With of the statements given above is/are correct? 3. Fish processed and canned
(a) 1 only (c) Both 1 and 2 4. Newspapers containing advertising material

(b) 2 only (d) Neither 1 nor 2 Which of the above items is/are exempted under GST
(Goods and Services Tax)?
3. The Chairmen of public sector banks are selected (a) 1 only (c) 1, 2 and 4 only
by the
(b) 2 and 3 only (d) 1, 2, 3 and 4
(a) Banks Board Bureau
9. With reference to the governance of public
(b) Reserve Bank of India
sector banking in India, consider the following
(c) Union Ministry of Finance statements:
(d) Management of concerned bank 1. Capital infusion into public sector banks by the
Government of India has steadily increased in
4. The Services Area Approach was implemented the last decade.
under the purview of 2. To put the public sector banks in order, the
(a) Integrated Rural Development Programme merger of associate banks with the parent State
Bank of India has been affected.
(b) Lead Bank Scheme
Which of the statements given above is/are correct?
(c) Mahatma Gandhi National Rural Employment
Guarantee Scheme (a) 1 only (c) Both 1 and 2

(d) National Skill Development Mission (b) 2 only (d) Neither 1 nor 2

6
www.iasscore.in ECONOMY WORKBOOK

10. Consider the following statements: 14. Which one of the following links all the ATMs in
1. Capital Adequacy Ratio (CAR) is the amount that India?
banks have to maintain in the form of their (a) Indian Banks’ Association
own funds to offset any loss that banks incur if
(b) National Securities Depository Limited
the accountholders fail to repay dues.
(c) National Payments Corporation of India
2. CAR is decided by each individual bank.
(d) Reserve Bank of India
Which of the statements given above is/are correct?
(a) 1 only (c) Both 1 and 2 15. With reference to India’s decision to levy an
equalization tax of 6% on online advertisement
(b) 2 only (d) Neither 1 nor 2
services offered by nonresident entities, which of
the following statements is/are correct?
11. Consider the following statements:
1. It is introduced as a part of the Income Tax Act.
1. The Reserve Bank of India manages and services
the Government of India Securities but not any 2. Nonresident entities that offer advertisement
State Government Securities. services in India can claim a tax credit in their
home country under the “Double Taxation
2. Treasury bills are issued by the Government of Avoidance Agreements”.
India and there are no treasury bills issued by
the State Governments. Select the correct answer using the code given below:

3. Treasury bills offer are issued at a discount from (a) 1 only (c) Both 1 and 2
the par value.
(b) 2 only (d) Neither 1 nor 2
Which of the statements given above is/are correct?
16. Consider the following statements:
(a) 1 and 2 only (c) 2 and 3 only
1. National Payments Corporation of India (NPCI)
(b) 3 Only (d) 1, 2 and 3 helps in promoting financial inclusion in the
country.
12. Which one of the following best describes the 2. NPCI has launched RuPay, a card payment
term “Merchant Discount Rate” sometimes seen in scheme.
news? Which of the statements given above is/are correct?
(a) The incentive given by a bank to a merchant (a) 1 only (c) Both 1 and 2
for accepting payments through debit cards
pertaining to that bank. (b) 2 only (d) Neither 1 nor 2
(b) The amount paid back by banks to their 17. Which of the following statements best describes
customers when they use debit cards for financial the- term ‘Scheme for Sustainable Structuring of
transactions for purchasing goods or services. Stressed Assets (S4A)’, recently seen in the news?
(c) The charge to a merchant by a bank for accepting (a) It is a procedure for considering the ecological
payments from his customers through the bank’s costs of developmental schemes formulated by
debit cards. the Government.
(d) The incentive is given by the Government to (b) It is a scheme of RBI for reworking the financial
merchants for promoting digital payments by structure of big corporate entities facing genuine
their customers through Point of Sale (PoS) difficulties.
machines and debit cards. (c) It is a disinvestment plan of the Government
regarding Central Public Sector Undertakings.
13. Which one of the following statements correctly
(d) It is an important provision in ‘The Insolvency
describes the meaning of legal tender money?
and Bankruptcy Code’ recently implemented by
(a) The money which is tendered in courts of law to the Government.
defray the fee of legal cases.
(b) The money which a creditor is under compulsion 18. What is the purpose of setting up of Small Finance
to accept in settlement of his claims. Banks (SFBs) in India?
1. To supply credit to small business units
(c) The bank money in the forms of cheque, drafts,
bills of exchange, etc. 2. To supply credit to small and marginal farmers
3. To encourage young entrepreneurs to set up
(d) The metallic money in circulation in a country. business particularly in rural areas.

7
ECONOMY WORKBOOK www.iasscore.in

Select the correct answer using the code given below: 23. The term ‘Core Banking Solution’ is sometimes such
(a) 1 and 2 only (c) 1 and 3 only in the news. Which of the following statements best
describes/describe this term?
(b) 2 and 3 only (d) 1, 2 and 3
1. It is a networking of a bank’s branches which
19. Which of the following is the most likely enables customers to operate their accounts
consequence of implementing the `Unified from any branch of the bank on its network
Payments Interface (UPI)’? regardless of where they open their accounts.

(a) Mobile wallets will not be necessary for online 2. It is an effort to increase RBI’s control over
payments. commercial banks through computerization.
3. It is a detailed procedure by which a bank with
(b) The digital currency will totally replace the
huge non-performing assets is taken over by
physical currency in about two decades.
another bank.
(c) FDI inflows will drastically increase.
Select the correct answer using the code given below:
(d) The direct transfer of subsidies to poor people
will become very effective. (a) 1 only (c) 1 and 3 only
(b) 2 and 3 only (d) 1, 2 and 3
20. What is/are the purpose/purposes of the
Government’s ‘Sovereign Gold Bond Scheme’ and 24. The establishment of “Payment Banks’ is being
‘Gold Monetization Scheme’? allowed in India to promote financial inclusion.
1. To bring the idle gold lying with India households Which of the following statements is/are correct in
into the economy this context?
2. To promote FDI in the gold and jewellery sector 1. Mobile telephone companies and supermarket
3. To reduce India’s dependence on gold imports chains that are owned and controlled by residents
are eligible to be promoters of Payment Banks.
Select the correct answer using the code given below
2. Payment Banks can issue both credit cards and
(a) 1 only (c) 1 and 3 only debit cards.
(b) 2 and 3 only (d) 1, 2 and 3 3. Payment Banks cannot undertake lending
activities.
21. The term ‘Base Erosion and profit shifting’ is
sometimes seen in the news in the context of: Select the correct answer using the code given below:

(a) Mining operation by multinational companies in (a) 1 and 2 only


resource-rich by backward areas (b) 1 and 3 only
(b) Curbing of the tax evasion by multinational (c) 2 only
companies
(d) 1, 2 and 3
(c) Exploitation of genetic resources of a country by
multinational companies 25. With reference to ‘Bitcoins’ sometimes seen in
the news, which of the following statements is/are
(d) Lack of consideration of environmental costs in
correct?
the planning of development projects
1. Bitcoins are tracked by the Central Banks of the
22. What is/are the purpose/purposes of the ‘Marginal countries.
Cost of Funds based Lending Rate (MCLR)’ 2. Anyone with a Bitcoin address can send and
announced by RBI? receive Bitcoins from anyone else with a Bitcoin
1. These guidelines help improve the transparency address.
in the methodology followed by banks for
3. Online payments can be sent without either side
determining the interest rates on advances
knowing the identity of the other.
2. These guidelines help ensure availability of
Select the correct answer using the code given below.
bank credit & interest rates which are fair to the
borrowers as well as the banks (a) 1 and 2 only
Select the correct answer using the code given below: (b) 2 and 3 only
(a) 1 only (c) Both 1 and 2 (c) 3 only
(b) 2 only (d) Neither 1 nor 2 (d) 1, 2 and 3

8
www.iasscore.in ECONOMY WORKBOOK

26. When the Reserve Bank of India reduces the 31. As regards the use of international food safety
Statutory Liquidity Ratio by 50 basis points, which standards as a reference point for the dispute
of the following is likely to happen? settlements, which one of the following does WTO
collaborate with?
(a) India’s GDP growth rate increases drastically
(a) Codex Alimentarius Commission
(b) Foreign Institutional Investors may bring more
capital into our country (b) International Federation of Standards Users

(c) Scheduled Commercial Banks may cut their (c) International Organization for Standardization
lending rates (d) World Standards Cooperation
(d) It may drastically reduce the liquidity to the
32. Which one of the following is not related to the
banking, system
United Nations?
27. ‘Basel III Accord’ or simply ‘Basel III’, often seen in (a) Multilateral Investment Guarantee Agency
the news, seeks to: (b) International Finance Corporation
(a) Develop national strategies for the conservation (c) International Centre for Settlement of Investment
and sustainable use of biological diversity Disputes
(b) Improve the banking sector’s ability to deal with (d) Bank for International Settlements
financial and economic stress and improve risk
management 33. The United Nations Framework Convention on
Climate Change (UNFCCC) is an international
(c) Reduce greenhouse gas emissions but places a treaty drawn at:
heavier burden on developed countries
(a) United Nations Conference on the Human
(d) Transfer technology from developed countries Environment, Stockholm,1972
to poor countries to enable them to replace the
use of chlorofluorocarbons in refrigeration with (b) UN Conference on Environment and Development,
Rio de Janeiro, 1992
harmless chemicals
(c) World Summit on Sustainable Development,
28. If the interest rate is decreased in an economy, it Johannesburg, 2002
will (d) UN Climate Change Conference, Copenhagen,
(a) Decrease the consumption expenditure in the 2009
economy
34. The International Development Association, a
(b) Increase the tax collection of the Government lending agency, is administrated by the:
(c) Increase the investment expenditure in the (a) International Bank for Reconstruction and
economy Development
(d) Increase the total savings in the economy (b) International Fund for Agricultural Development
(c) United Nations Development Programme
29. What is/are the facility/facilities the beneficiaries
can get from the services of Business Correspondent (d) United Nations Industrial Development
(Bank Saathi) in branchless areas? Organization
1. It enables the beneficiaries to draw their 35. Consider the following statements:
subsidies and social security benefits in their
1. The Commonwealth has no charter, treaty or
villages.
constitution
2. It enables the beneficiaries in rural areas to
2. All the territories/countries once under the
make deposits and withdrawals.
British empire (jurisdiction/rule/mandate)
Select the correct answer using the code given below: automatically joined the Commonwealth as its
members
(a) 1 only (c) Both 1 and 2
Which of the statements given above is/are correct?
(b) 2 only (d) Neither 1 nor 2
(a) 1 only (c) Both 1 and 2
30. The terms ‘Marginal Standing Facility Rate’ and (b) 2 only (d) Neither 1 nor 2
‘Net Demand and Time Liabilities’, sometimes
appearing in news, are used in relation to: 36. “Gold Tranche” (Reserve Tranche) refers to
(a) Banking operations (a) a loan system of the World Bank
(b) Communication networking (b) one of the operations of a Central Bank
(c) Military strategies (c) a credit system granted by WTOs to its members
(d) Supply and demand of agricultural products (d) a credit system granted by IMF to its members

9
ECONOMY WORKBOOK www.iasscore.in

37. In the context of the Indian economy non-financial give the Central Government the right to issue
debt includes which of the following? directions to the RBI in public interest.
1. Housing loans owed by households 3. The Governor of the RBI draws his power from
2. Amounts outstanding on credit cards the RBI Act.
3. Treasury bills Which of the above statements are correct?
Select the correct answer using the code given below: (a) 1 and 2 only (c) 1 and 3 only
(a) 1 only (c) 3 only (b) 2 and 3 only (d) 1, 2 and 3
(b) 1 and 2 only (d) 1, 2 and 3
42. With reference to urban cooperative banks in
38. If the RBI decides to adopt an expansionist India consider the following statements:
monetary policy, which of the following would it
1. They are supervised and regulated by local
not do?
boards set up by the state governments
1. Cut and optimize the Statutory Liquidity Ratio
2. They can issue equity shares and preference
2. Increase the Marginal Standing Facility Rate
shares.
3. Cut the Bank Rate and Repo Rate
3. They were brought under the purview of
Select the correct answer using the code given below: the Banking Regulation Act, 1949 through an
(a) 1 and 2 only (c) 1 and 3 only Amendment in 1966.
(b) 2 only (d) 1, 2 and 3 Which of the statements given above is/are correct?
(a) 1 only (c) 1 and 3 only
39. Consider the following statements:
1. In terms of short-term credit delivery to the (b) 2 and 3 only (d) 1, 2 and 3
agriculture sector, District Central Cooperative
Banka (DCCB) delivers more credit in comparison 43. Indian Government Bond Yields are influenced by
to Scheduled Commercial Banks and Regional which of the following?
Rural Banks. 1. Actions of the United States Federal Reserve
2. One of the most important functions of DCCBs 2. Actions of the Reserve Bank of India
is to provide funds to the Primary Agricultural
Credit Societies. 3. Inflation and short-term interest rates

Which of the statements given above is / are correct? Select the correct answer using the code given below.

(a) 1 only (c) Both 1 and 2 (a) 1 and 2 only (c) 3 only

(b) 2 only (d) Neither 1 nor 2 (b) 2 only (d) 1, 2 and 3

40. With reference to the Indian economy, consider 44. Which one of the following effects of creation of
the following statements. black money in India has been the main cause of
worry to the Government of India?
1. “Commercial Paper’ is a short-term unsecured
promissory note. (a) Diversion of resources to the purchase of real
2. ‘Certificate of Deposit’ is a long term instrument estate and investment in luxury housing
issued by the Reserve Bank of India to a (b) Investment in unproductive activities and
corporation. purchase of precious stones, jewelley, gold, etc.
3. ‘Call Money’ is a short-term finance used for (c) Large donations to political parties and growth of
interbank transactions. regionalism
4. ‘Zero-Coupon Bonds’ are the interest bearing (d) Loss of revenue to the state exchequer due to tax
short-term bonds issued by the Scheduled evasion
Commercial Banks to corporations.
Which of the statements given above is/are correct? 45. Which one of the following is likely to be the most
inflationary in its effects?
(a) 1 and 2 only (c) 1 and 3 only
(a) Repayment of public debt
(b) 4 only (d) 2, 3 and 4 only
(b) Borrowing from the public to finance a budget
41. Consider the following statements: deficit
1. The Governor of the Reserve Bank of India (RBI) (c) Borrowing from the banks to finance a budget
is appointed by the Central Government. deficit

2. Certain provisions in the Constitution of India (d) Creation of new money to finance a budget deficit

10
www.iasscore.in ECONOMY WORKBOOK

46. The money multiplier in an economy increases 50. Which one of the following situations best reflects
with which one of the following? “Indirect Transfers” often talked about in media
(a) Increase in the cash Reserve Ration in the banks recently with reference to India?

(b) Increase in the Statutory Liquidity Ratio in the (a) An Indian company investing in a foreign
banks enterprise and paying taxes to the foreign country
on the profits arising out of its investment.
(c) Increase in the banking habit of the people
(d) Increase in the population of the country (b) A foreign company investing in India and paying
taxes to the country of its base on the profits
47. With reference to India, consider the following arising out of its investment.
statements: (c) An Indian company purchases tangible assets in
1. Retail investors through demat account can a foreign country and sells such assets after their
invest in ‘Treasury Bills’ and ‘Government of value increases and transfers the proceeds to
India Debt Bonds’ in primary market. India.
2. The ‘Negotiated Dealing System-Order Matching’
(d) A foreign company transfers shares and such
is a government securities trading platform of
the Reserve Bank of India. shares derive their substantial value from assets
located in India.
3. The ‘Central Depository Services Ltd’. is jointly
promoted by the Reserved Bank of India and the
51. With reference to the Indian economy, consider
Bombay Stock Exchange.
the following statements:
Which of the statements given above is/are correct?
1. A share of the household financial savings goes
(a) 1 only (c) 3 only towards government borrowings.
(b) 1 and 2 (d) 2 and 3 only 2. Dated securities issued at market-related rates
in auctions form a large component of internal
48. In India, the central bank’s function as the ‘lender debt.
of last resort’ usually refers to which of the
following? Which of the above statements is/are correct?
1. Lending to trade and industry bodies when they (a) 1 only (c) Both 1 and 2
fail to borrow from other sources
(b) 2 only (d) Neither 1 nor 2
2. Providing liquidity to the banks having a
temporary crisis 52. Consider the following statements:
3. Lending to governments to finance budgetary 1. Tight monetary policy of US Federal Reserve
deficits could lead to capital flight.
Select the correct answer using the code given below. 2. Capital flight may increase the interest cost
(a) 1 and 2 of firms with existing External Commercial
Borrowings (ECBs).
(b) 2 only
3. Devaluation of domestic currency decreases the
(c) 2 and 3 only
currency risk associated with ECBs.
(d) 3 only
Which of the statements given above are correct?
49. With reference to the India economy, what are the (a) 1 and 2 only (c) 1 and 3 only
advantages of “Inflation-Indexed Bonds (IIBs)”?
(b) 2 and 3 only (d) 1, 2 and 3
1. Government can reduce the coupon rates on its
borrowing by way of IIBs. 53. With reference to the ‘Banks Board Bureau (BBB),
2. IIGs provide protection to the investors from which of the following statements are correct ?
uncertainty regarding inflation. 1. The Governor of RBI is the Chairman of BBB.
3. The interests received as well as capital gains on 2. BBB recommends for the selection of heads for
IIBs are not taxable. Public Sector Banks.
Which of the statements given above are correct? 3. BBB helps the Public Sector Banks in developing
(a) 1 and 2 only strategies and capital raising plans.
(b) 2 and 3 only Select the correct answer using the code given below:
(c) 1 and 3 only (a) 1 and 2 only (c) 1 and 3 only
(d) 1, 2 and 3 (b) 2 and 3 only (d) 1, 2 and 3

11
ECONOMY WORKBOOK www.iasscore.in

54. With reference to Convertible Bonds, consider the (b) Expenditure Management Commission
following statements: (c) Financial Stability and Development Council
1. As there is an option to exchange the bond for (d) Reserve Bank of India
equity, Convertible Bonds pay a lower rate of
interest. 56. With reference to Non-Fungible Tokens (NFTs),
2. The option to convert to equity affords the consider the following statements:
bondholder a degree of indexation to rising 1. They enable the digital representation of
consumer prices. physical assets.
Which of the statements given above is/are correct? 2. They are unique cryptographic tokens that exist
on a blockchain.
(a) 1 only (c) Both 1 and 2
3. They can be traded or exchanged at equivalency
(b) 2 only (d) Neither 1 nor 2 and therefore can be used as a medium of
commercial transactions.
55. In India, which one of the following is responsible
for maintaining price stability by controlling Which of the statements given above are correct?
inflation? (a) 1 and 2 only (c) 1 and 3 only 1
(a) Department of Consumer Affairs (b) 2 and 3 only (d) 1, 2 and 3

ECONOMIC SECTORS

1. Consider the following statements: 3. With reference to India’s projects on connectivity,


Statement-I: consider the following statements:
1. East-West Corridor under Golden Quadrilateral
India, despite having uranium deposits, depends on
Project connects Dibrugarh and Surat.
coal for most of its electricity production.
2. Trilateral Highway connects Moreh in Manipur
Statement-II:
and Chiang Mai in Thailand via Myanmar.
Uranium, enriched to the extent of at least 60%, is 3. Bangladesh - China -India - Myanmar Economic
required for the production of electricity.
Corridor connects Varanasi in Uttar Pradesh
Which one of the following is correct in respect of the with Kunming in China.
above statements?
How many of the above statements are correct?
(a) Both Statement-I and Statement-II are correct
(a) Only one (c) All three
and Statement-II is the correct explanation for
Statement-I (b) Only two (d) None
(b) Both Statement-I and Statement-II are correct
and Statement-II is not the correct explanation 4. About three-fourths world’s cobalt, a metal
for Statement-I required for the manufacture of batteries for
electric motor vehicles, is produced by:
(c) Statement-I is correct Statement-II is incorrect
(a) Argentina
(d) Statement-I is incorrect Statement-II is correct
(b) Botswana
2. Consider the following infrastructure sectors: (c) the Democratic Republic of the Congo
1. Affordable housing
(d) Kazakhstan
2. Mass rapid transport
3. Health rare 5. Consider the following pairs:
4. Renewable energy Port Well known as
On how man of the above does UNOPS Sustainable 1. Kamarajar Port : First major port in India
Investments in Infrastructure and Innovation (S3i) registered as a company
initiative focus for its investments? 2. Mundra Port : Largest privately owned
(a) Only one port in India
(b) Only two 3. Visakhapatnam Port : Largest container port in
India
(c) Only three
How many of the above pairs are correctly matched?
(d) All four

12
www.iasscore.in ECONOMY WORKBOOK

(a) Only one pair 9. The economic cost of food grains to the Food
(b) Only two pairs Corporation of India is Minimum Support Price
and bonus (if any) paid to the farmers plus
(c) All three pairs
(a) Transportation cost only
(d) None of the pairs
(b) Interest cost only
6. Consider the following statements: (c) Procurement incidentals and distribution cost
The ‘Stability and Growth Pact’ of the European Union (d) Procurement incidentals and charges for godowns
is a treaty that
1. Limits the levels of budgetary deficit of the 10. Consider the following statements:
countries of the European Union. 1. Most of India’s external debt is owed by
2. Makes the countries of the European Union to governmental entities.
share their infrastructure facilities. 2. All of India’s external debt is denominated in US
3. Enables the countries of the European Union to dollars.
share their technologies.
Which of the statements given above is/are correct?
4. How many of the above statements are correct?
(a) 1 only (c) Both 1 and 2
(a) Only one
(b) 2 only (d) Neither 1 nor 2
(b) Only two
(c) All three 11. In the context of India, which of the following
(d) None factors is/are contributor/contributors to reducing
the risk of a currency crisis?
7. Consider the following statement: 1. The foreign currency earnings of India’s IT sector
Statement – I: 2. Increasing the government expenditure
India accounts for 3.2% of global export of goods. 3. Remittances from Indians abroad
Statement-II: Select the correct answer using the code given below.
Many local companies and some foreign companies (a) 1 only (c) 2 only
operating in India have taken advantage of India’s (b) 1 and 3 only (d) 1, 2 and 3
‘Production-linked Incentive’ scheme.
Which one of the following is correct in respect of the 12. Which of the following is issued by registered
above statements? foreign portfolio investors to overseas investors
who want to be part of the Indian stock market
(a) Both statement-I and Statement-II are correct
without registering themselves directly?
and Statement-II is the correct explanation for
Statement-I (a) Certificate of Deposit
(b) Both Statement-I and Statement-II are correct (b) Commercial Paper
and Statement-II is not the correct explanation (c) Promissory Note
for Statement-I
(d) Participatory Note
(c) Statement-I is correct but Statement-II is incorrect.
(d) Statement-I- is incorrect but Statement-II is 13. With reference to India’s Five-Year Plans, which of
correct. the following statements is/are correct?
1. From the Second Five-Year Plan, there was a
8. Consider the following statements with reference
determined thrust towards substitution of basic
to India:
and capital good industries.
1. According to the ‘Micro, Small and Medium
2. The Fourth Five-Year Plan adopted the objective
Enterprises Development (MSMED) Act, 2006’,
of correcting the earlier trend of increased
the ‘medium enterprises’ are those with
concentration of wealth and economic power.
investments in plant and machinery between Rs.
15 crore and Rs. 25 crore. 3. In the Fifth Five-Year Plan, for the first time, the
2. All bank loans to the Micro, Small and Medium financial sector was included as an integral part
Enterprises qualify under the priority sector. of the Plan.

Which of the statements given above is/are correct? Select the correct answer using the code given below.

(a) 1 only (c) Both 1 and 2 (a) 1 and 2 only (c) 2 only

(b) 2 only (d) Neither 1 nor 2 (b) 3 only (d) 1, 2 and 3

13
ECONOMY WORKBOOK www.iasscore.in

14. Consider the following statements: 6. Sesamum


1. Purchasing Power Parity (PPP) exchange rates 7. Turmeric
are calculated by comparing the prices of the
same basket of goods and services in different The Cabinet Committee on Economic Affairs has
countries. announced the Minimum Support Price for which of
the above?
2. In terms of PPP dollars, India is the sixth largest
economy in the world. (a) 1, 2, 3 and 7 only

Which of the statements given above is/are correct? (b) 2, 4, 5 and 6 only

(a) 1 only (c) Both 1 and 2 (c) 1, 3, 4, 5 and 6 only

(b) 2 only (d) Neither 1 nor 2 (d) 1, 2, 3, 4, 5, 6 and 7

15. Among the agricultural commodities imported by 20. With reference to the ‘National Investment and
India, which one of the following accounts for the Infrastructure Fund’, which of the following
highest imports in terms of value in the last five statements is/are correct?
years? 1. It is an organ of NITI Aayog.
(a) Spices (c) Pulses 2. It has a corpus of Rs. 4,00,000 crore at present.
(b) Fresh fruits (d) Vegetable oils Select the correct answer using the code given below:
(a) 1 only (c) Both 1 and 2
16. With reference to land reforms in independent
India, which one of the following statements is (b) 2 only (d) Neither 1 nor 2
correct?
21. Consider the following statements:
(a) The ceiling laws were aimed at family holdings
1. The Standard Mark of Bureau of Indian
and not individual holdings.
Standards (BIS) is mandatory for automotive
(b) The major aim of land reforms was providing tyres and tubes.
agricultural land to all the landless. 2. AGMARK is a quality Certification Mark issued
(c) It resulted in cultivation of cash crops as a by the Food and Agriculture Organisation (FAO).
predominant form of cultivation. Which of the statements given above is/are correct?
(d) Land reforms permitted no exemptions to the (a) 1 only (c) Both 1 and 2
ceiling limits. (b) 2 only (d) Neither 1 nor 2

17. Consider the following statements: 22. What is/are the advantage/advantages of
1. Coal sector was nationalized by the Government implementing the `National Agriculture Market’
of India under Indira Gandhi. scheme?
2. Now, coal blocks are allocated on lottery basis. 1. It is a pan-India electronic trading portal for
3. Till recently, India imported coal to meet the agricultural commodities.
shortages of domestic supply, but now India is 2. It provides the farmer’s access to nationwide
self- sufficient in coal production. market, with prices commensurate with the
quality of their produce.
Which of the statements given above is/are correct?
Select the correct answer using the code given below:
(a) 1 only (c) 3 only
(a) 1 only (c) Both 1 and 2
(b) 2 and 3 only (d) 1, 2 and 3
(b) 2 only (d) Neither 1 nor 2
18. Among the following, which one is the largest
exporter of rice in the world in the last five years? 23. With reference to `Quality Council of India (QCI)
consider the following statements:
(a) China (c) Myanmar
1. QCI was set up jointly by the Government of
(b) India (d) Vietnam India and the Indian Industry.
19. Consider the following: 2. The chairman of QCI is appointed by the Prime
Minister on the recommendations of the industry
1. Areca nut
to the Government.
2. Barley
Which of the above statements is/are correct?
3. Coffee
(a) 1 only (c) Both 1 and 2
4. Finger millet
(b) 2 only (d) Neither 1 nor 2
5. Groundnut

14
www.iasscore.in ECONOMY WORKBOOK

24. Recently, India’s first ‘National Investment and 30. Which of the following brings out the ‘Consumer
Manufacturing Zone’ was proposed to be set up in: Price Index Number for Industrial Workers?
(a) Andhra Pradesh (c) Maharashtra (a) The Reserve Bank of India
(b) Gujarat (d) Uttar Pradesh (b) The Department of Economic Affairs

25. With reference to ‘Pradhan Mantri Fasal Bima (c) The Labour Bureau
Yojana’, consider the following statements: (d) The Department of Personnel and Training
1. Under this scheme, farmers have to pay a
uniform premium of two percent for any crop 31. Consider the following statements:
they cultivate in any season of the year. 1. The Accelerated Irrigation Benefits Programme
2. This scheme covers post-harvest losses arising was launched during 1996-97 to provide loan
out of cyclones and unseasonal rains. assistance to poor farmers.
Which of the statements given about is/are correct? 2. The Command Area Development Programme
(a) 1 only (c) Both 1 and 2 was launched in 1974-75 for the development of
water-use efficiency.
(b) 2 only (d) Neither 1 nor 2
Which of the statements given above is/are correct?
26. The following is the purpose of ‘UDAY’, a scheme of (a) 1 only (c) Both 1 and 2
the Government?
(a) Providing technical and financial assistance to (b) 2 only (d) Neither 1 nor 2
start-up entrepreneurs in the field or renewable
source of energy. 32. With reference to the Indian Renewable Energy
Development Agency Limited (IREDA), which of
(b) Providing electricity to every household in the
the following statements is/are correct?
country by 2018
1. It is a Public Limited Government Company.
(c) Replacing the coal-based power plants with
natural gas, nuclear, solar, wind and tidal power 2. It is a Non-Banking Financial Company.
plants over a period of time. Select the correct answer using the code given below:
(d) Providing for financial turnaround and revival of (a) 1 only (c) Both 1 and 2
power distribution companies
(b) 2 only (d) Neither 1 nor 2
27. In the ‘Index of Eight Core Industries’, which one of
the following is given the highest weight? 33. In India, the steel production industry requires the
(a) Coal production import of:

(b) Electricity generation (a) Saltpetre (c) Coking coal

(c) Fertilizer production (b) Rock phosphate (d) All of the above
(d) Steel production
34. What are the significances of a practical approach
to sugarcane production known as the ‘Sustainable
28. The Fair and Remunerative Price (FRP) of
sugarcane is approved by the: Sugarcane Initiative’?

(a) Cabinet Committee on Economic Affairs 1. Seed cost is very low in this compared to the
conventional method of cultivation.
(b) Commission for Agricultural Costs and Prices
2. Drip irrigation can be practiced very effectively
(c) Directorate of Marketing and Inspection, Ministry in this.
of Agriculture
3. There is no application of chemical/ inorganic
(d) Agricultural Produce Market Committee fertilizers at all in this.
29. In, India, markets in agricultural products are 4. The scope for intercropping is more in this
regulated under the: compared to the conventional method of
cultivation.
(a) Essential Commodities Act, 1955
Select the correct answer using the code given below:
(b) Agricultural Produce Market Committee Act
enacted by States. (a) 1 and 3 only
(c) Agricultural Produce (Grading and Marking) Act, (b) 1, 2 and 4 only
1937
(c) 2, 3 and 4 only
(d) Food Products Order, 1956 and Meat and Food
(d) 1, 2, 3 and 4
Products Order, 1973

15
ECONOMY WORKBOOK www.iasscore.in

35. In the context of the affairs of which of the following 3. To provide emerging and eco-friendly food
is the phrase “Special Safeguard Mechanisms” processing technologies to entrepreneurs.
mentioned in the news frequently?
Select the correct answer using the codes given below:
(a) United Nations Environment Programme
(a) 1 only (c) 2 and 3 only
(b) World Trade Organization
(b) 1 and 2 only (d) 1, 2 and 3
(c) ASEAN- India Free Trade Agreement
(d) G-20 Summits 40. Consider the following statements:
1. The Union Government fixes the Statutory
36. What is/ are the recent policy initiative(s) of the Minimum Price of sugarcane for each sugar
Government of India to promote the growth of season
manufacturing sector?
2. Sugar and sugarcane are essential commodities
1. Setting up of National Investment and
under the Essential Commodities Act.
Manufacturing Zones.
2. Providing the benefit of single window clearance. Which of the statements given above is/are correct?
3. Establishing the Technology Acquisition and (a) 1 only (c) Both 1 and 2
Development Fund. (b) 2 only (d) Neither 1 nor 2
Select the correct answer using the codes given below:
41. Which one of the following is not a feature of
(a) 1 only (c) 1 and 3 only
Limited Liability Partnership firm?
(b) 2 and 3 only (d) 1, 2 and 3
(a) Partners should be less than 20
37. In India the overall Index of Industrial Production, (b) Partnership and management need not be
the Indices of Eight Core Industries has combined separate
weight of 37.90%. Which of the following is among
those Eight Core Industries? (c) Internal governance may be decided by mutual
agreement among partners
1. Cement
2. Fertilizers (d) It is corporate body with perpetual succession
3. Natural Gas
42. The SEZ Act, 2005 which came into effect in
4. Refinery products
February 2006 has certain objectives. In this
5. Textiles context, consider the following:
Select the correct answer using the codes given below: 1. Development of infrastructure facilities
(a) 1 and 5 only (c) 1, 2, 3 and 4 only 2. Promotion of investment from foreign sources
(b) 2, 3 and 4 only (d) 1, 2, 3, 4 and 5 3. Promotion of exports of services only
Which of the above are the objectives of this Act?
38. Despite having large reserves of coal, why does
India import millions of tonnes of coal? (a) 1 and 2 only (c) 2 and 3 only
1. It is the policy of India to save its own coal (b) 3 only (d) 1, 2 and 3
reserves for the future and import it from other
countries for the present use. 43. With reference to the international trade of India
2. Most of the power plants in India are coal-based at present, which of the following statements is/are
and they are not able to get sufficient supplies of correct?
coal from within the country. 1. India’s merchandise exports are less than its
3. Steel companies need a large quantity of coking merchandise imports.
coal which has to be imported.
2. India’s imports of iron and steel, chemicals,
Which of the statements given above is/are correct? fertilizers and machinery have decreased in
(a) 1 only (c) 1 and 3 only recent years,
(b) 2 and 3 only (d) 1, 2 and 3 3. India’s exports of services are more than its
imports of services.
39. With what purpose is the Government of India 4. India suffers from an overall trade/current
promoting the concept of “Mega food parks”? account deficit.
1. To provide good infrastructure facilities for the Select the correct answer using the code given below:
food processing industry.
(a) 1 and 2 only (c) 3 only
2. To increase the processing of perishable items
and reduce wastage. (b) 2 and 4 only (d) 1, 3 and 4 only

16
www.iasscore.in ECONOMY WORKBOOK

44. The term ‘West Texas Intermediate’, sometimes 3. Production of cement


found in news, refers to a grade of: Select the correct answer using the code given below:
(a) Crude oil (c) Rare earth elements (a) 1 and 2 only (c) 1 and 3 only
(b) Bullion (d) Uranium (b) 2 and 3 only (d) 1, 2 and 3

45. With reference to Trade-Related Investment 49. With reference to pulse production in India,
Measures (TRIMS), which of the following consider the following statements:
statements is/are correct? 1. Black gram can be cultivated as both kharif and
1. Quantitative restriction on imports by foreign rabi crop.
investors is prohibited. 2. Green-gram alone accounts for nearly half of
2. They apply to investment measures related to pulse production.
trade in both goods and services. 3. In the last three decades, while the production
3. They are not concerned with the regulation of of kharif pulses has increased, the production of
foreign investment. rabi pulses has decreased.

Select the correct answer using the code given below: Which of the statements given above is/are correct?

(a) 1 and 2 only (c) 1 and 3 only (a) 1 only (c) 2 only

(b) 2 only (d) 1, 2 and 3 (b) 2 and 3 only (d) 1, 2 and 3

46. In India, which of the following can be considered 50. In the context of India, which of the following is/
are considered to be practice(s) of eco-friendly
as public investment in agriculture?
agriculture?
1. Fixing Minimum Support Price for agricultural
1. Crop diversification
produce of all crops
2. Legume intensification
2. Computerization of Primary Agricultural Credit
Societies 3. Tensiometer use

3. Social Capital Development 4. Vertical farming

4. Free electricity supply to farmers Select the correct answer using the code given below:

5. Waiver of agricultural loans by the banking (a) 1, 2 and 3 only (c) 4 only
system (b) 3 only (d) 1, 2, 3 and 4
6. Setting up of cold storage facilities by the
governments. 51. What are the advantages of fertigation in
agriculture?
Select the correct answer using the code given below:
1. Controlling the alkalinity of irrigations water is
(a) 1, 2 and 5 only (c) 2, 3 and 6 only possible.
(b) 1, 2, 4 and 5 only (d) 1, 2, 3, 4, 5 and 6 2. Efficient application of Rock Phosphate and all
other phosphatic fertilizers is possible.
47. Consider the following statements: 3. Increased availability of nutrients to plants is
1. The value of Indo-Sri Lanka trade has consistently possible.
increased in the last decade 4. Reduction in the leaching of chemical nutrients
2. “Textile and textile articles” constitute and is possible.
important item of trade between India and Select the correct answer using the code given below:
Bangladesh.
(a) 1, 2 and 3 only (c) 1, 3 and 4 only
3. In the last five years, Nepal has been the largest
(b) 1, 2 and 4 only (d) 2, 3 and 4 only
trading partner of India in South Asia.
Which of the statements given above is/are correct? 52. Consider the following:
(a) 1 and 2 only (c) 3 only 1. Foreign currency convertible bonds.

(b) 2 only (d) 1, 2 and 3 2. Foreign institutional investment with certain


conditions
48. Steel slag can be the material for which of the 3. Global depository receipts
following? 4. Non-resident external deposits
1. Construction of base road Which of the above can be included in Foreign Direct
2. Improvement of agricultural soil Investments?

17
ECONOMY WORKBOOK www.iasscore.in

(a) 1, 2 and 3 only (c) 2 and 4 only 57. Consider the following:
(b) 3 only (d) 1 and 4 only 1. Asian Infrastructure Investment Bank
2. Missile Technology Control Regime
53. Consider the following statements: 3. Shanghai Cooperation Organisation
The effect of devaluation of a currency is that it India is a member of which of the above?
necessarily
(a) 1 and 2 only (c) 2 and 3 only
1. Improves the competitiveness of the domestic
(b) 3 only (d) 1, 2 and 3
exports in the foreign markets
2. Increases the foreign value of domestic currency 58. Consider the following statements:
3. Improves the trade balance 1. Vietnam has been one of the fastest growing
economies in the world in the recent years.
Which of the above statement is/are correct?
2. Vietnam is led by a multi-party political system.
(a) 1 only (c) 3 only
3. Vietnam’s economic growth is linked to its
(b) 1 and 2 only (d) 2 and 3 only integration with global supply chains and focus
on exports.
54. With reference to foreign-owned e-commerce 4. For a long time Vietnam’s low labour costs and
firms operating in India, which of the following stable exchange rates have attracted global
statements is/are correct? manufacturers.
1. They can sell their own goods in addition to 5. Vietnam has the most productive e-service sector
offering their platforms as market-places. in the Indo-Pacific region.
2. The degree to which they can own big sellers on Which of the statements given above are correct?
their platforms is limited. (a) 2 and 4 (c) 1, 3 and 4
Select the correct answer using the code given below: (b) 3 and 5 (d) 1 and 2
(a) 1 only (c) Both 1 and 2
59. In India, which one of the following compiles
(b) 2 only (d) Neither 1 nor 2 information on industrial disputes, closures,
entrenchments and lay-offs in factories employing
55. With reference to the “G20 Common Framework”, worker?
consider the following statements: (a) Central Statistics Office
1. It is an initiative endorsed by the G20 together (b) Department for Promotion of Industry and
with the Paris Club. Internal Trade
2. It is an initiative to support Low Income (c) Labour Bureau
Countries with unsustainable debt.
(d) National Technical Manpower Information
Which of the statements given above is/are correct? System
(a) 1 only (c) Both 1 and 2
60. In India, what is the role of the Coal Controller’s
(b) 2 only (d) Neither 1 nor 2 Organization (CCO) ?
1. CCO is the major source of Coal Statistics in
56. With reference to the expenditure made by an Government of India.
organization or a company, which of the following
2. It monitors progress of development of Captive
statements is/are correct? Coal/Lignite blocks.
1. Acquiring new technology is capital expenditure. 3. It hears any objection to the Government’s
2. Debt financing is considered capital expenditure, notification relating to acquisition of coal-
while equity financing is considered revenue bearing areas.
expenditure. 4. It ensures that coal mining companies deliver
the coal to end users in the prescribed time.
Select the correct answer using the code given below:
Select the correct answer using the code given below:
(a) 1 only (c) Both 1 and 2
(a) 1, 2 and 3 only (c) 1 and 2 only
(b) 2 only (d) Neither 1 nor 2
(b) 3 and 4 only (d) 1, 2 and 4 only

18
www.iasscore.in ECONOMY WORKBOOK

SOCIO-ECONOMIC SECTORS

1. Which one of the following best describes the (c) Statement-I is correct but Statement-II is incorrect
concept of ‘Small Farmer Large Field’? (d) Statement-I is incorrect but Statement-II is correct
(a) Resettlement of a large number of people,
uprooted from their countries due to war, by 4. Consider the following statements:
giving them a large cultivable land which they 1. The Self-Help Group (SHG) programme was
cultivate collectively and share the produce originally initiated by the State Bank of India
(b) Many marginal farmers in an area organize by providing microcredit to the financially
themselves into groups and synchronize and deprived.
harmonize selected agricultural operations 2. In an SHG, all members of a group take
(c) Many marginal farmers in an area together make responsibility for a loan that an individual
member takes.
a contract with a corporate body are surrender
their land to the corporate body of a fixed term 3. The Regional Rural Banks and Scheduled
for which the 6.onDorate body makes a payment Commercial Banks support SHGs.
of agreed amount to the farmers How many of the above statements are correct
(d) A company extends loans, technical knowledge (a) Only one
and material inputs to a number of small
farmers in an area so that they produce the (b) Only two
agricultural commodity required by the company (c) All three
for its manufacturing process and commercial
(d) None
production
5. In the context of any country, which one of the
2. Consider the following markets: following would be considered as part of its social
1. Government Bond Market capitals?
2. Call Money Market (a) The proportion of literates in the population
3. Treasury Bill Market (b) The stock of its buildings, other infrastructure
4. Stock Market and machines
How many of the above are included in capital (c) The size of population in the working age group
markets?
(d) The level of mutual trust and harmony in the
(a) Only one society
(b) Only two
6. In a given year in India, official poverty lines are
(c) Only three
higher in some States than in others because
(d) All four
(a) Poverty rates vary from State to State
3. Consider the following statements: (b) Price levels vary from State to State
Statement-I: (c) Gross State Product varies from State to State
India’s public sector health care system largely focuses
(d) Quality of public distribution varies from State to
on curative care with limited preventive, promotive
and rehabilitative care. State

Statement-II: 7. With reference to the provisions made under the


Under India’s decentralized approach to health care National Food Security Act, 2013, consider the
delivery, the States are primarily responsible for following statements:
organizing health services.
1. The families coming under the category of
Which one of the following is correct in respect of the ‘below poverty line (BPL)’ only are eligible to
above statements?
receive subsidized food grains
(a) Both Statement-I and Statement-II are correct
2. The eldest woman in a household, of age 18 years
and Statement-II is the correct explanation for
Statement-I or above, shall be the head of the Household for
the purpose of issuance of a ration card.
(b) Both Statement-I and Statement-II are correct
and Statement-II is not the correct explanation 3. Pregnant women and lactating mothers are
for Statement-I entitled to a ‘takehome ration’ of 1600 calories

19
ECONOMY WORKBOOK www.iasscore.in

per day during pregnancy and for six months 12. Regarding ‘Atal Pension Yojana’, which of the
thereafter. following statements is/are correct?
Which of the statements given above is/are correct? 1. It is a minimum guaranteed pension scheme
mainly targeted at unorganized sector workers
(a) 1 and 2 only (c) 1 and 3 only
2. Only one member of a family can join the scheme
(b) 2 only (d) 3 only
3. Same amount of pension is guaranteed for the
8. With reference to Pradhan Mantri Kaushal Vikas spouse for life after the subscriber’s death.
Yojana, consider the following statements: Select the correct answer using the code given below:
1. It is the flagship scheme of the Ministry of Labour (a) 1 only (c) 1 and 3 only
and Employment.
(b) 2 and 3 only (d) 1, 2 and 3
2. It, among other things, will also impart training
in soft skills, entrepreneurship, financial and
13. Pradhan Mantri MUDRA Yojana is aimed at:
digital literacy.
3. It aims to align the competencies of the (a) Bringing the small entrepreneurs into the formal
unregulated workforce of the country to the financial system
National Skill Qualification Framework. (b) Providing loans to poor farmers for cultivating
Which of the statements given above is/are correct? particular crops

(a) 1 and 3 only (c) 2 and 3 only (c) Providing pensions to old and destitute persons

(b) 2 only (d) 1, 2 and 3 (d) Funding the voluntary organization involved
in the promotion of skill development and
9. Consider the following statements: employment generation
The nation-wide ‘Soil Health Card Scheme’ aims at:
14. ‘SWAYAM’, an initiative of the Government of
1. expanding the cultivable area under irrigation. India, aims at:
2. enabling the banks to assess the quantum of (a) Promoting Self Help Groups in rural areas
loans to be granted to farmers on the basis of soil
quality. (b) Providing financial and technical assistance to
3. checking the overuse of fertilizers in farmlands. young start-up entrepreneurs

Which of the above statements is/are correct? (c) Promoting the education and health of adolescent
girls
(a) 1 and 2 only (c) 2 and 3 only
(d) Providing affordable and quality education to the
(b) 3 only (d) 1, 2 and 3 citizens for free
10. Who among the following can join the National 15. With reference to the ‘Stand Up India Scheme’,
Pension System (NPS)? which of the following statements is/are correct?
(a) Resident Indian citizens only 1. Its purpose is to promote entrepreneurship
(b) Persons of age from 21 to 55 only among SC/ST and women entrepreneurs.
(c) All-State Government employees joining the 2. It provides for refinancing through SIDBI.
services after the date of notification by the Select the correct answer using the code given below.
respective State Governments
(a) 1 only
(d) All Central Governments Employees including
those of Armed Forces joining the services on or (b) 2 only
after 1st April 2004 (c) Both 1 and 2

11. ‘Recognition of Prior Learning Scheme’ is (d) Neither 1 nor 2


sometimes mentioned in the news with reference
to: 16. “Pradhan Mantri Jan-Dhan Yojna’ has been
launched for
(a) Certifying the skills acquired by construction
workers through traditional channels. (a) Providing housing loan to poor people at cheaper
interest rates
(b) Enrolling the persons in Universities for distance
learning programs. (b) Promoting women’s Self-Help Groups in
backward areas
(c) Reserving some skilled jobs to rural and urban
poor in some public sector undertakings. (c) Promoting financial inclusion in the country
(d) Certifying the skills acquired by trainees under (d) Providing financial help to the marginalized
the National Skill Development Programme. communities

20
www.iasscore.in ECONOMY WORKBOOK

17. Disguised unemployment generally means: 21. The Multi-Dimensional Poverty Index developed
(a) A large number of people remain unemployed by Oxford Poverty and Human Development
Initiative with UNDP support covers which of the
(b) Alternative employment is not available following?
(c) Marginal productivity of labour is zero 1. Deprivation of education, health, assets, and
(d) Productivity of workers is low services at household level
2. Purchasing power parity at the national level
18. Which of the following can be said to be essentially
3. Extent of the budget deficit and GDP growth rate
the parts of Inclusive Governance?
at national level
1. Permitting the Non-Banking Financial
Select the correct answer using the codes given below:
Companies to do banking
2. Establishing effective District Planning (a) 1 only (c) 1 and 3 only
Committees in all the districts (b) 2 and 3 only (d) 1, 2 and 3
3. Increasing government spending on public
health 22. The endeavor of Janini Suraksha yojana program
is
4. Strengthening the Mid-day Meal Scheme
1. To promote institutional deliveries
Select the correct answer using the codes given below:
2. To provide monetary assistance to the mother to
(a) 1 and 2 only meet the cost of delivery
(b) 3 and 4 only 3. To provide for wage loss due to pregnancy and
confinements
(c) 2, 3 and 4 only
Which of the above are correct?
(d) 1, 2, 3 and 4
(a) 1 and 2 only (c) 3 only
19. With reference to the National Rural Health
(b) 2 only (d) 1, 2 and 3
Mission, Which of the following are the jobs of
‘ASHA”, trained community health workers?
23. Consider the following:
1. Accompanying women to the health facility for
1. Hotels and restaurants
antenatal care checkup
2. Motor Transport undertakings
2. Using pregnancy test kits for early detection
pregnancy 3. Newspaper Establishments

3. Providing information on nutrition and 4. Private Medical Institutions


immunization The Employers of which of the above can have the
4. Conducting the delivery of baby. “Social Security coverage under Employees State
Insurance Scheme?
Select the correct answer using the codes given below:
(a) 1, 2 and 3 only (c) 1, 3 and 4 only
(a) 1, 2 and 3 only (c) 1 and 3 only
(b) 4 only (d) 1, 2, 3 and 4
(b) 2 and 3 only (d) 1, 2, 3 and 4
24. How do District Rural Development Agencies
20. How does the National Rural Livelihood Mission (DRDAs) help in the reduction of rural poverty in
seek to improve Livelihood Mission seeks to India?
improve livelihood options of rural poor? 1. DRDAs act as Panchayati Raj Institutions in
1. By setting up a large number of new certain specified backward regions of the
manufacturing industries and agribusiness country.
centers in rural areas. 2. DRDAs undertake an area-specific scientific
study of the cause of poverty and malnutrition
2. By strengthening ‘self-help groups’ and providing
and prepare detailed remedial measures.
skills development
3. DRDAs secure inter-sectoral and inter-
3. By supplying seeds, fertilizers, diesel pump-sets departmental coordination and cooperation for
and micro-irrigation equipment free of cost of the effective implementation of anti-poverty
farmers. programs.
Select the correct answer using the codes given below: 4. DRDAs watch over and ensure effective
utilization of the funds intended for anti-poverty
(a) 1 and 2 only (c) 1 and 3 only program.
(b) 2 only (d) 1, 2 and 3 Which of the statements given above is/ are correct?

21
ECONOMY WORKBOOK www.iasscore.in

(a) 1, 2 and 3 only (c) 4 only 2. Purchase of combine harvesters, tractors and
(b) 3 and 4 only (d) 1, 2, 3 and 4 mini trucks
3. Consumption requirements of farm households
25. Among the following who are eligible to benefit 4. Post-harvest expenses
from the “Mahatma Gandhi national rural
employment guarantee act”? 5. Construction of family house and setting up of
village cold storage facility
(a) Adult members of only the scheduled caste and
scheduled tribe households. Select the correct answer using the code given below:
(b) Adult members of below poverty line (BPL) (a) 1, 2 and 5 only (c) 2, 3, 4 and 5 only
households. (b) 1, 3 and 4 only (d) 1, 2, 3, 4 and 5
(c) Adult members of households of all backward
communities. 30. Consider the following statements:
(d) Adult members of any household. 1. In the case of all cereals, pulses and oil-seeds, the
procurement at Minimum Support Price (MSP) is
26. Inclusive growth as enunciated in the Eleventh-Five unlimited in any State/UT of India.
Year Plan does not include one of the following: 2. In the case of cereals and pulses, the MSP is fixed
(a) Reduction of poverty in any State/UT at a level to which the market
(b) Extension of employment opportunities price will never rise.

(c) Strengthening of capital market Which of the statements given above is/are correct?
(a) 1 only (c) Both 1 and 2
(d) Reduction of gender inequality
(b) 2 only (d) Neither 1 nor 2
27. An objective of the National Food Security
Mission is to increase the production of certain 31. With reference to casual workers employed in
crops through area expansion and productivity India, consider the following statements:
enhancement in a sustainable manner in the 1. All casual workers are entitled for Employees
identified districts of the country. What are those Provident Fund coverage.
crops? 2. All casual workers are entitled for regular
(a) Rice and wheat only working hours and overtime payment.
(b) Rice, wheat, and pulses only 3. The government can by a notification specify
that an establishment or industry shall pay
(c) Rice, wheat, pulses, and oilseeds only wages only through its bank account.
(d) Rice, wheat, pulses, oilseeds, and vegetables Which of the above statements are correct?
28. Which of the following factors/policies were (a) 1 and 2 only (c) 1 and 3 only
affecting the price of rice in India in the recent (b) 2 and 3 only (d) 1, 2 and 3
past?
1. Minimum Support Price 32. Which of the following activities constitute real
2. Government’s trading sector in the economy?
3. Government’s stockpiling 1. Farmers harvesting their crops
4. Consumer subsidies 2. Textile mills converting raw cotton into fabrics
Select the correct answer using the code given below 3. A commercial bank lending money to a trading
company
(a) 1, 2 and 4 only (c) 2 and 3 only
4. A corporate body issuing Rupee Denominated
(b) 1, 3 and 4 only (d) 1, 2, 3 and 4
Bonds overseas
29. Under the Kisan Credit Card Scheme, short term Select the correct answer using the code given below:
credit support is given to farmers for which of the
(a) 1 and 2 only (c) 1, 3 and 4 only
following purposes?
1. Working capital for maintenance of farm assets (b) 2, 3 and 4 only (d) 1, 2, 3 and 4

22
ECONOMY
ANSWERS

BASIC CONCEPTS

1. Exp. (d) 4. Exp. (c)


Capital Output Ratio (ICOR) Interest rate and investment
 Capital Output Ratio (ICOR) measures the  The relationsip between the interest rate and
percentage increase in capital formation required
investment Expenditure is illustrated by the
obtaining a percentage increase in GDP.
investment curve of the economy.
 Entrepreneurs, by investing their own savings and
informally mobilising the savings of their friends  The curve has a downward slope, indicating that
and relatives contribute to the process of capital a drop in interest rate, causes the investment-
formation. spending to rise.
 These informal funding supplements the funds  The marginal efficiency of capital states that for
made available by the formal means of raising investment to be worthwhile, it needs to give a
resources from banks, financial institutions and
higher rate of return than the interest rate.
capital markets.
 If capital to output ratio is high then capital 5. Exp. (c)
formation may not result in significant increase in
Capital formation
the output.
 Capital formation takes place when a country does
2. Exp. (c) not spend all its current income and consumption,
Opportunity cost but saves a part of it and uses it for investment for
 Opportunity cost is the cost of choosing one increasing further production.
alternative over another and missing the benefit  This act of saving and investment is described as
offered by the forgone opportunity, investing or
capital accumulation or capital formation.
otherwise.
 Opportunity cost refers to a benefit that a person 6. Exp. (b)
could have received, but gave up, to take another
1st STAGE
course of action. Stated differently, an opportunity
cost represents an alternative given up when a  This stage has been called high population growth
decision is made. Opportunity cost is also called potential stage. It is characterised by high and
the economic cost. fluctuating birth and death rates which will almost
3. Exp. (b) neutralize each other.

Venture Capital 2nd STAGE


 Venture Capital is a kind of private equity, a  In this stage the death rate is decreasing while the
kind of financing provided to small-size, early- birth rate remains constant at a high level.
stage, emergent firms (often called start-ups) for
3rd STAGE
long term that show or have proven high growth
prospective in terms of annual revenue or the  In this stage, birth rate as compared to the death
number of employees or both. rate declines more rapidly. As a result, population
 Venture Capital funds invest in such start-ups in grows at a diminishing rate.
exchange for equity. They get ownership stake in
the companies they invest in. Generally, the early- 7. Exp. (d)
stage company are based on some innovative Microfinance
technology. They usually belong to high technology
industries such as information technology,  Financial inclusion emerging as a major policy
biotechnology or social media. objective in the country, Microfinance has

23
ECONOMY WORKBOOK www.iasscore.in

occupied center stage as a promising conduit for India’s labour productivity grew by over 14 per
extending financial services to unbanked sections cent every year. But between financial years of
of population 2011 and 2015, this rate fell to just half of that (7.4
 Microcredit, or microfinance, is banking the per cent) and continued its deceleration to just 3.7
unbankable, bringing credit, savings and other per cent between financial years of 2016 and 2018.
essential financial services such as insurance and  Statement 2 is incorrect: The rate of growth in
fund transfer facilities within the reach of millions population and workforce during the last four
of people who are too poor to be served by regular decade in rural areas have always been lower
banks, in most cases because they are unable to than of the urban areas.
offer sufficient collateral.  Statement 3 is correct: Rural non-farm economy, in
recent times, is considered as an effectual strategy
8. Exp. (b) for decentralization of economic activities to rural
Greendex scores India. The Economic Census of India estimates that
around 41.89 million rural people are employed in
 The Greendex scores each respondent based on the
non-agricultural establishments which registered
consumption patterns they report in the survey a growth rate of 4.56 % during 1998-2005.
and compares scores of average consumers within
each country in environmentally sustainable  As per the 2011 Census, 68.8 per cent of country’s
population and 72.4 per cent of workforce resided
manner.
in rural areas. However, steady transition to
 The Greendex measures consumer behavior in urbanization over the years is leading to the
four broad areas: housing, transportation, food decline in the rural share in population, workforce
consumption, and goods. and GDP of the country. So, statement 4 is correct.

9. Exp. (a) 13. Exp. (a)


 Option 1 is correct: Not depending on short term  The interest coverage ratio measures a company’s
foreign borrowings and focusing on long term ability to handle its outstanding debt. It is one of a
plans will give immunity to the economy. number of debt ratios that can be used to evaluate
a company’s financial condition.
 Option 3 is incorrect: India’s rupee is a partially
convertible currency—rupees can be exchanged  Statements 1 and 2 are correct: It is one of the
at market rates in certain cases, but approval parameters that help understand and evaluate
is required for larger amounts. Making the present and emerging risks of a firm a bank is
rupee a fully convertible currency would mean lending to.
increased liquidity in financial markets, improved  Statement 3 is incorrect: A higher ratio indicates
employment and business opportunities, and a better financial health as it means that the
easy access to capital. Some of the disadvantages company is more capable to meeting its interest
include higher volatility, an increased burden of obligations from operating earnings.
foreign debt, and an effect on the balance of trade 14. Exp. (a)
and exports.
 Statement 1 is correct: The weightage of food in
10. Exp. (d) the CPI is close to 50%. Weightage of Food group in
WPI is 39.06%.
 M3 = Currency in circulation + Demand & Time
deposits in Bank  Statement 2 is correct: The WPI inflation does not
capture price changes of services but the CPI does.
11. Exp. (b)
 Statement 3 is incorrect: The Reserve Bank of India
 Foreign direct investment (FDI) takes place when on 1 April 2014 adopted the Consumer Price Index
investor establishes foreign business operation (CPI) as the key measure of Inflation. Till now the
or acquires foreign business aseets , including Reserve Bank of India (RBI) was using Wholesale
establishing ownership or controlling interest in a Price Index (WPI) to gauge and measure indicative
foreign company. Thus, Foreign direct investment inflation projections.
(FDI), being a non debt capital flow, is a leading 15. Exp. (b)
source of external finance.
Rapid Financing Instrument (RFI)
12. Exp. (b)
 The Rapid Financing Instrument (RFI) provides
 Statement 1 is incorrect: Productivity is a measure rapid financial assistance, which is available to
of the efficiency with which resources, both all member countries facing an urgent balance
human and material, are converted into goods of payments need. The RFI was created as part
and services. There is wide disparity in worker of a broader reform to make the IMF’s financial
productivity between rural and urban areas. support more flexible to address the diverse needs

24
www.iasscore.in ECONOMY WORKBOOK

of member countries. The RFI replaced the IMF’s  Statement 2 is incorrect: An increase in REER
previous emergency assistance policy and can be implies that exports become more expensive and
used in a wide range of circumstances. imports become cheaper; therefore, an increase
Rapid Credit Facility (RCF) indicates a loss in trade competitiveness.
 The Rapid Credit Facility (RCF) provides rapid  Statement 3 is correct: NEER is the weighted
concessional financial assistance to low-income
geometric average of the bilateral nominal
countries (LICs) facing an urgent balance of
payments (BoP) need with no ex post conditionality exchange rates of the home currency in terms
where a full-fledged economic program is neither of foreign currencies. The REER is the weighted
necessary nor feasible. The RCF was created average of NEER adjusted by the ratio of domestic
under the Poverty Reduction and Growth Trust price to foreign prices. Increasing trend in
(PRGT) as part of a broader reform to make the domestic inflation relative to inflation in other
Fund’s financial support more flexible and better
countries creates a divergence in NEER and REER.
tailored to the diverse needs of LICs, including in
times of crisis. There are three windows under 17. Exp. (b)
RCF: (i) a “regular window” for urgent BoP
needs caused by wide range of sources including  Statement 1 is incorrect: If the inflation is high
domestic instability, emergencies and fragility; RBI tries to reduce the liquidity from the market,
(ii) an “exogenous shock window” for urgent BoP by selling Government securities to the public via
needs caused by a sudden, exogenous shock; and
open market operation.
(iii) a “large natural disaster window” for urgent
BoP needs arising from natural disasters where  Statement 2 is correct: Rupee depreciation
damage is assessed to be equivalent to or exceed means, fall in value of rupee with respect to dollar.
20 percent of the member’s GDP. Access under the In free floating exchange rate regime, depreciation
RCF is subject to annual and cumulative limits, with
takes place when the demand for dollar is more
higher access limits applying for the large natural
disaster window. For higher income countries that than the supply.thus, RBI is likely to sell dollars in
are non-PRGT eligible, a similar Rapid Financing the economy to increase the supply of the dollar.
Instrument (RFI) is available.  Statement 3 is correct: If the interest rate in US
16. Exp. (c) and EU falls, there will be an inflow of dollars
in the Indian market, leading to appreciation of
 Statement 1 is correct: NEER is a measure of
value of a currency against a weighted average of the rupee. To reduce the supply of dollar in the
several foreign currencies. An increase in NEER economy, RBI will like to buy the dollars from the
indicates appreciation of rupee. market.

ECONOMIC MEASUREMENTS

1. Exp. (b) per cent weight to the demographic performance


criterion.
 The Fifteenth Finance Commission (XVFC)’s ToR
was unique and wide ranging in many ways.  XVFC has re-introduced tax effort criterion to
The Commission was asked to recommend reward fiscal performance.
performance incentives for States in many areas
like power sector, adoption of DBT, solid waste Criteria Weight(%)
management etc. Another unique ToR was to
recommend funding mechanism for defence and Population 15.0
internal security. Area 15.0
 Horizontal devolution: Based on principles of
Forest & ecology 10.0
need, equity and performance, overall devolution
formula is as follows: Income distance 45.0
 On horizontal devolution, while XVFC agreed that
Tax & Fiscal efforts 2.5
the Census 2011 population data better represents
the present need of States, to be fair to, as well Demographic performance 12.5
as reward, the States which have done better on
the demographic front, XVFC has assigned a 12.5 Total 100

25
ECONOMY WORKBOOK www.iasscore.in

2. Exp. (a) develops that shows the stock’s openness to the


market risk.
 Option a is correct: Sterilization usually takes
the form of an open market operation, in which  This helps the investor to decide whether he wants
a central bank sells or purchases government to go for the riskier stock that is highly correlated
bonds on an open market in the amount it with the market (beta above 1), or with a less
purchases or sells foreign currency on the foreign volatile one (beta below 1).
exchange market, so that the amount of domestic  Beta is the key factor used in the Capital Asset
currency in circulation remains unchanged. Price Model (CAPM) which is a model that
The open market operation effectively offsets or measures the return of a stock.
sterilizes the impact of the intervention on the
monetary base.  The volatility of the stock and systematic risk
can be judged by calculating beta. A positive beta
What is Sterilization? value indicates that stocks generally move in the
Sterilization is a form of monetary action in which same direction with that of the market and the
a central bank seeks to limit the effect of inflows and vice versa.
outflows of capital on the money supply. Sterilization
5. Exp. (c)
most frequently involves the purchase or sale of
financial assets by a central bank and is designed to Human Capital Formation
offset the effect of foreign exchange intervention.  Human Capital Formation refers to stock of ‘skill
and expertise’ embodied in humans. Human
3. Exp. (d) capital is as important as physical capital for
 Statement 1 is incorrect: any dividend and economic development.
Interest income from InvITs is completely taxable  Human capital formation includes expenditure
as per the slab rate of the investor. on education, on health and on on-job-
training(intangible) are key instruments of human
 Statement 2 is correct: In an 11 February 2021
capital formation, also implies the development
statement, the Finance Ministry announced that of abilities and skills, knowledge among the
it would be introducing relevant amendments to: population of the country.
 The Securities Contracts (Regulation) Act (SCRA)  In order to develop various sectors of the economy,
1956; the country should introduce manpower planning
 The Securitisation and Reconstruction of for the development of its human resources.
Financial Assets and Enforcement of Security
Interest (SARFAESI) Act 2002; and 6. Exp. (c)

 The Recovery of Debts Due to Banks and Financial Economic development


Institutions Act (‘Recovery of Debts Act’) 1993.  Economic development includes not only economic
growth but also various other economic changes
 Such amendments aim to augment further funding
that improve the quality of life or standard of
for the infrastructure and real estate sectors by living of people in a country.
enabling infrastructure investment trusts (InvITs)
 If with economic growth, a country experiences
and real estate investment trusts (REITs) to easily
various economic changes such as a reduction in
avail debt financing from investors, including poverty and unemployment, reduction in income
foreign portfolio investors (FPIs). and wealth inequality, increase in literacy rate,
 With InvITs and REITs now recognized as improvement in health and hygiene, etc, that
borrowers under the SARFAESI Act, lenders improve the quality of life then that is economic
development.
to these trusts shall have adequate statutory
enforcement options, absence of which was  If gains of increase in per capita income are grabbed
earlier becoming a constraint for bankers to lend by a small section of society, then economic growth
directly at trust level. will not lead to economic development.

4. Exp. (d) 7. Exp. (c)

 Beta is a numeric value that measures the  NSSO 70th Round “Situation Assessment Survey of
fluctuations of a stock to changes in the overall Agricultural Households
stock market. Beta measures the responsiveness  Rajasthan has the highest agricultural households
of a stock’s price to changes in the overall stock (78.4%) among rural households, in terms of
market. proportion, in absolute numbers it is Uttar
 On comparison of the benchmark index for e.g. Pradesh.
NSE Nifty to a particular stock returns, a pattern  The share of OBC households is about 45 percent.

26
www.iasscore.in ECONOMY WORKBOOK

 In Kerala, about 61 percent of agricultural families 13. Exp. (a)


have more income from agricultural sources.
Inflation
8. Exp. (b)  Inflation is that it causes the value of the currency
Indian economy to decline over time.
 Real GDP means GDP at a constant price. GDP  Inflation lets debtors pay lenders back with money
growth is not steadily increased in last decade that is worth less than it was when they originally
2005-2015 because of recession which happened borrowed it.
in 2007-08.  A rise in the inflation rate will tend to cause bond
 Gross domestic product at market prices is the sum prices to drop. Inflation behaves similarly to bond
of the gross values added of all resident producers yields, moving in the opposite direction from bond
at market prices, plus taxes less subsidies on prices.
imports.
14. Exp. (b)
 GDP at market price has continuously increased in
last decade. Capital gains
 Capital means asset, capital gains means a rise in
9. Exp. (c)
the value of asset.
Controlling inflation  Sale increase is not asset price rise, property value
 Controlling inflation is responsibility of RBI, rise is asset value rise, similar is the case with
central and state government. painting
 RBI control inflation through monetary policy  Capital gain is a rise in the value of a capital asset
 Inflation can be controlled by decreasing money (investment or real estate) that gives it a higher
supply. worth than the purchase price. The gain is not
realized until the asset is sold. A capital gain may
 Increased money circulation increases inflation.
be short-term (one year or less) or long-term.
10. Exp. (d)
15. Exp. (b)
Inflation
High food inflation
 General Price rise may be caused by an increase in
the money supply as the real value of the money  Food price inflation has remained persistently
reduces. elevated, reflecting in part the structural demand-
supply mismatch in several commodities.
 A decrease in the aggregate level of output will
lead to rise in the general level of prices  The trend of food inflation was pointing out at
 Effective demand also increases when there is less not only structural demand-supply mismatch in
commodities comprises the essential consumption
purchasing power due to inflation.
basket but also changing consumption basket.
11. Exp. (d)  The various explanations for high food inflation
New money that unfavorable weather conditions did play a
role in food prices, structural factors were also
 New money into the economy will increases the
significant.The stubbornly high food prices have
money supply which will be inflationary.
raised a number of questions about inefficient
 In case deficit financing without creating money marketing systems, weak storage infrastructure
the money supply remains the same. and stagnant productivity which affects supply
 Fresh money creates a more inflationary effect chain.
than money in circulation.
16. Exp. (c)
12. Exp. (a)
Base effect
National income
 The base effect refers to the impact of the rise
 NNP at Factor Cost( Net National Product at Factor
Cost) is the net money value of all the goods in the price level (i.e. last year’s inflation) in the
and services produces by normal residents of a previous year over the corresponding rise in price
country. levels in the current year (i.e., current inflation).
 It includes income of Indian citizens whether living  If the price index had risen at a high rate in the
in or outside India. It is net of the national income corresponding period of the previous year leading
which means, it do not include depreciation. to a high inflation rate, some of the potential rises

27
ECONOMY WORKBOOK www.iasscore.in

is already factored in, therefore a similar absolute  A monetary stimulus will see the central bank
increase in the Price index in the current year will expanding money supply or reducing interest
lead to relatively lower inflation rates. rates to encourage consumer spending.
 On the other hand, if the inflation rate was too low  A fiscal stimulus is one in which the government
in the corresponding period of the previous year, spends more from its own pocket.
even a relatively smaller rise in the Price Index  In fiscal stimulus govt slashes tax rates and may
will arithmetically give a high rate of current increases subsidies but not abolish subsidies, it
inflation. puts more money in the hands of consumers and
17. Exp. (d) spending goes up – thereby encouraging demand
& growth.
Growth rate and Per capita income
 Growth rates of GDP and Per capita income from 22. Exp. (c)
2006-07 to 2010-11 are not steady.
 Before 2009 WPI was available on weakly basis,
 Due to the global financial crisis and consequently from 2009 it was changed to monthly basis.
recessionary and deflationary tendencies in global
economics.  Weight of Food Items
 WPI – food articles =15.40 %, food products =
18. Exp. (b) 11.50%
Inflation  CPI for Industrial Workers = 57 %
 Due to an increase in wealth, the spending
increases. 23. Exp. (b)

 As the spending increases, Demand increases Gross Domestic Product (GDP)


leading to increasing in price causing Inflation.  It is the total monetary or market value of all the
finished goods and services produced within a
19. Exp. (c) country’s borders in a specific time period. As a
 Deflation is the economic term used to describe broad measure of overall domestic production,
the drop in prices for goods and services. it functions as a comprehensive scorecard of the
 Deflation slows down economic growth. It country’s economic health.
normally takes place during times of economic  Though GDP is usually calculated on an annual
uncertainty when there is demand for goods basis, it can be calculated on a quarterly basis as
and services is lower, along with higher levels of well by CSO.
unemployment.
 GDP in 2000-2001 was 19.2 lakh crores which
 When prices fall, the inflation rate drops below 0 increased to 58 lakh crores in 2009-2010
percent.
 Which is nearly 3 times.
20. Exp. (b)  With the rapid growth of the private sector after
Meltdown LPG share of public sector in GDP gone down.
 Meltdown means weakness in financial market 24. Exp: (b)
which affects investment
In Economic Recession, the Government can:
 Economic meltdown follows normally a black
swan event and quickly results in financial assets  Decrease interest rate to make more room for
losing in value in a tailspin, often resulting in borrowing
liquidity crisis.  Increase Government Spending
Recession  Cut Tax rates to increase spending capacity of
 Recession is a slowdown or a massive contraction people
in economic activities. A significant fall in spending
generally leads to a recession. It means negative 25. Exp: (a)
growth in GDP for two consecutive quarters but in The main influences on buying plans that change
general it is a fall in GDP. demand are:
Slow down  Prices of related goods
 It means reduction in growth rate in GDP.  Income Expectations
21. Exp. (a)  Number of buyers

Fiscal stimulus  Preferences

 A ‘fiscal stimulus’ is an attempt by government to The Demand increases when


kickstarts a sluggish economy through a package  The demand for a good increases, if the price of
of measures. one of its substitutes rises.

28
www.iasscore.in ECONOMY WORKBOOK

 The demand for a good increases, if the price of of the Securities and Exchange Board of India Act,
one of its substitutes rises. 1992.
 An inferior good is an economic term that  There are a total of six credit agencies in India viz,
describes a good whose demand drops when CRISIL, CARE, ICRA, SMREA, Brickwork Rating,
people’s incomes rise. and India Rating and Research Pvt. Ltd.
 If the price of a good falls, the quantity demanded  Statement 2 is correct: ICRA Limited is a public
of that good increases. limited company that was set up in 1991 in
Gurugram. The company was formerly known as
26. Exp: (a) Investment Information and Credit Rating Agency
 Rising interest rates will deter people to borrow of India Limited.
money and hence it will not aid in demand pull  Statement 3 is correct: Brickwork Ratings is
inflation recognised as external credit assessment agency
(ECAI) by Reserve Bank of India (RBI) to carry out
27. Exp. (b)
credit ratings in India.
 Statement 1 is incorrect: Credit Rating Agencies
 Brickwork Rating was established in 2007 and is
(CRA) analyzes a debtor’s ability to repay the debt
promoted by Canara Bank. It offers ratings for
and also rate their credit risk.
bank loans, SMEs, corporate governance rating,
 All the credit rating agencies in India are regulated municipal corporation, capital market instrument,
by SEBI (Credit Rating Agencies) Regulations, 1999 and financial institutions.

MONEY AND BANKING

1. Exp. (a)  To mitigate stress in currency and bond markets,


many emerging market central banks used foreign
 Option 1 is correct: Condition during the
exchange interventions, and for the first time,
pandemic: During the pandemic, central banks in
both advanced and emerging market economies asset purchase programs.
took unprecedented measures to ease financial  More recently, in response to rapidly growing
conditions and support the economic recovery, inflation, central banks around the world have
including interest-rate cuts and asset purchases. tightened monetary policy by increasing interest
 Condition post pandemic: With inflation at multi- rates.
decade highs in many countries and pressures 2. Exp. (c)
broadening beyond food and energy prices,
policymakers have pivoted toward tighter policy,  Statement 1 is correct: Alternatives to SWIFT and
where central banks in many emerging markets the U.S. dollar are coming from two directions:
proactively started to hike rates, followed by their cryptocurrencies and central bank digital
counterparts in advanced economies in the final currencies. Cryptocurrencies (or “crypto”) are
months of 2021. a form of payment that can circulate without
the need for a central monetary authority such
 Option 2 is correct: Central banks use monetary
as a government or bank and are created using
policy to manage economic fluctuations and
achieve price stability, which means that distributed ledger technologies and cryptographic
inflation is low and stable. Central banks in many techniques that enable people to buy, sell or trade
advanced economies set explicit inflation targets. them securely in a decentralized way. These
Central bank assumes that they have the ability decentralized networks are controlled by no one
to counteract the rising consumer prices via and enable privacy from government intrusion or
monetary policy. intervention.
 Statement 2 is correct: Currently, access to digital
Reason why option 2 is a correct explanation for central bank money is limited to central bank
option no. 1: operating hours, traditionally less than 24 hours
a day and usually five days a week.8 CBDCs could
 In response to the COVID-19 pandemic, central be available 24 hours a day and seven days a week
banks took actions to ease monetary policy, or only during certain specified times (such as the
provide liquidity to markets, and maintain the
operating hours of largevalue payment systems).
flow of credit.

29
ECONOMY WORKBOOK www.iasscore.in

CBDC could be available permanently or for a  Increase in SLR decrease money multiplier, as it
limited duration (eg it could be created, issued and reduces flow of funds back to market
redeemed on an intraday basis).  Population is not related to money multiplier.
3. Exp. (a) 8. Exp. (c)
Bank Board Bureau (BBB) Exempt supply
 BBB is autonomous body of Central Government.  As per section 2(47) of CGST Act 2017, an exempt
 It was announced by Union Government in August supply means any goods or services or both which
2015 as part of seven point Indradhanush Mission attract nil rate of tax or that may be wholly exempt
to revamp PSBs and started functioning in April from
2016.
 tax under section 11 or may be wholly exempt
 It had replaced Appointments Board of
from tax under section 6 of IGST act or including
Government.
non-taxable supply.
 It is housed in Reserve Bank of India’s (RBI) central
office in Mumbai, Maharashtra.  Cereals, eggs, fish etc. are exempted from GST,
but not ‘cooked’ and ‘processed’ which will be
 It comprises eminent professionals and officials available for sale in restaurants.
for public sector banks (PSBs).
 Newspapers containing advertisements will be
Functions of BBB
taxed under GST with rate of 5% on revenue with
 Give recommendations for appointment of full- availing ITC for tax paid by advertisement agency
time Directors as well as non-Executive Chairman
on commission received.
of PSBs.
 Give advice to PSBs in developing differentiated 9. Exp. (b)
strategies for raising funds through innovative
Public sector banking
financial methods and instruments and to deal
with issues of stressed assets.  After the Securitisation and Reconstruction of
 Guide banks on mergers and consolidations and Financial Assets and Enforcement of Securities
governance issues to address bad loans problem Interest Act, 2002 (also known as the SARFAESI
among other issues. Act), the government capital infusion steadily
decreased until NPA problem introduced in 2015.
4. Exp. (b)  The merger of SBI bank with its associated banks
 Lead bank scheme Service Area Approach under under section 35 of SBI ACT 1935 will result in
which villages are identified and assigned to bank creation of a stronger merged entity, this will
branches based on their proximity and contiguity. minimize vulnerability.
Sub Service Area is a cluster of more than one
village wherein up to 1000 to 1500 households are 10. Exp. (a)
brought under such cluster for providing banking  Capital Adequacy Ratio is known capital-to-risk
services.
weighted assets ratio is used for the protection of
5. Exp. (b) depositors and promotion of stability of financial
systems across the world.
 Deposits of banks are liability not assets
 Capital Adequacy Ratio is a measure of the capital
6. Exp. (d) of the bank which is expressed as a percentage of
 To stop slide of rupee against dollar, we need to bank’s risk-weighted credit exposures.
improve inflow of forex and increase its supply  It is decided by central banks and bank regulators
and control domestic currency money supply. on recommendation of BASEL 3 norms to prevent
 Is a correct step as it will bring more forex (commercial) banks from taking excess leverage
 Is a correct step as it will bring more forex and becoming insolvent.
through FPI
11. Exp. (c)
 Is a correct step as it will bring more forex
through Debt  RBI is Banker / Debt manager for both State and
Union Government.
 Is not a correct step, as it increases money
supply  According to Department of Economic Affairs,

7. Exp. (b)  Short-term debt of the Central Government


on residual maturity basis includes 14-day
 Increase in CRR decrease money multiplier, as it intermediate treasury bills, regular treasury bills,
reduces flow of funds back to market dated securities maturing in the ensuing one year
 It increases money multiplier by increasing the and external debt with remaining maturity of less
velocity of money than one year.

30
www.iasscore.in ECONOMY WORKBOOK

 Short-term debt of State Governments comprises  It provides that income of the non-resident from
internal debt that includes market loans maturing the provision of the specified services to the
within next one year, and repayment of loans to asses under chapter VIII of the FA is exempt from
Centre. income tax in the hands of the non-resident if the
same is chargeable to equalization levy.
 T-bills are sold at discount and re-purchased at par
 However, it does not mean that the income of the
value (face value).
non-resident from the specified services would be
12. Exp. (c) charged to income tax if the same is not chargeable
to equalization levy.
Merchant discount rate
 Merchant discount rate on debit cards is the 16. Exp. (c)
amount that a merchant has to pay to its service National Payments Corporation of India (NPCI)
providers when a consumer swipes her card on
 National Payments Corporation of India (NPCI),
the merchant’s pointofsales terminal. It is also
an umbrella organisation for operating retail
applicable for online transactions and QRbased
payments and settlement systems in India, is an
transactions. initiative of Reserve Bank of India (RBI) and Indian
 The amount that the merchant pays for every Banks’ Association (IBA) under the provisions of
transaction gets distributed among three the Payment and Settlement Systems Act, 2007,
stakeholders—the bank that enables the for creating a robust Payment & Settlement
transaction, the vendor that installs the PoS Infrastructure in India.
machine and the card network provider such as  NPCI launched the Rupay card in 2012, as 7th
Visa or MasterCard. payment gateway in the world.

13. Exp. (b) 17. Exp. (b)

Legal tender money Scheme for Sustainable Structuring of Stressed


Assets (S4A):
 Legal tender money is fiat money that cannot be
refused by any citizen of the country for settlement  The RBI had launched the S4A scheme to help
of any kind of transaction. resolve the corporate debt problem and strengthen
the lenders’ ability to deal with stressed assets.
 The ‘Legal tender’ is the money that is recognized
 Under this scheme, a company’s debt is bifurcated
by the law of the land, as valid for payment of
into two parts sustainable and unsustainable
debt. It must be accepted for discharge of debt.
based on the cash flows of the company’s project.
The RBI Act of 1934, which gives the Central Bank
the sole right to issue banknotes, states that “Every  The sustainable debt cannot be less than 50% of
banknote shall be legal tender at any place in India existing debt and will have to be serviced over the
same terms as that of existing facilities.
in payment for the amount expressed therein”.
 The unsustainable debt can be converted into
14. Exp. (c) equity, Optionally Convertible Debentures or
Redeemable Optionally Convertible Preference
National Payments Corporation of India: Share with clearly spelled out terms.
 National Financial Switch (NFS) is the largest
network of shared automated teller machines 18. Exp. (a)
(ATMs) in India. Small Finance Banks (SFBs)
 It was designed, developed and deployed by  Small finance banks are a type of niche banks in
the Institute for Development and Research in India. Banks with a small finance bank license can
Banking Technology (IDRBT) in 2004, with the goal provide basic banking service of acceptance of
of inter-connecting the ATMs in the country and deposits and lending.
facilitating convenience banking.  SFBs was recommended by the Nachiket Mor
 It is run by the National Payments Corporation of committee on financial inclusion.
India(NPCI).  The objectives of setting up of small finance banks
will be to further financial inclusion by
15. Exp. (d)  The provision of savings vehicles
Equalization tax:  Supply of credit to small business units; small and
 The levy was introduced in the Budget as part of marginal farmers; micro and small industries; and
the finance bill and not as a part of the Income Tax other unorganized sector entities, through high
Act. technology-low cost operations.
 So, because of this the companies would not be  However, apart from the provision of credit, there
able to take the benefit of tax treaties to avoid is no explicit provision that SFBs have been set up
double taxation in their home countries. to encourage setting of businesses in rural areas

31
ECONOMY WORKBOOK www.iasscore.in

19. Exp. (a)  Marginal Cost of Funds based Lending Rate


(MCLR) improves the transmission of policy rates
 Mobile wallets will not be necessary, because your
into the lending rates of banks. These measures
mobile number is directly linked with the bank
are expected to improve transparency in the
account using bank’s ‘customized app’ which is
methodology followed by banks for determining
built on the UPI platform.
interest rates on advances.
 It was launched in April 2016 by National Payment
 The guidelines are also expected to ensure the
Corporation of India and in the last two years, the
platform has emerged as a popular choice among availability of bank credit at interest rates which
users for sending and receiving money. are fair to the borrowers as well as the banks.
Further, marginal cost pricing of loans will help
 UPI is a payment system that allows money the banks become more competitive and enhance
transfer between any two bank accounts by using their long-run value and contribution to economic
a smartphone.
growth.
20. Exp. (c)
23. Exp. (a)
 Gold Monetization Scheme and Gold Monetization
Core Banking Solutions (CBS)
Scheme
 A solution that enables banks to offer a multitude
 Sovereign Gold Bond Scheme’ and ‘Gold
of customer-centric services on a 24×7 basis from
Monetization Scheme’ aims to bring the idle gold
lying with India households into the economy a single location.
which is tangible material to make more useful in  Using CBS, customers can access their accounts
economy. from any branch, anywhere, irrespective of where
 Sovereign Gold Bond Scheme’ and ‘Gold they have physically opened their accounts. Almost
Monetization Scheme’ not promote any FDI in gold all branches of commercial banks, including the
and jewellery sector. Regional Rural Banks (RRBs), are brought into the
core-banking fold.
 It will reduce gold imports by reducing demand
for physical gold by shifting a part of the demand  By Core Banking Solution no any change in RBI’s
for physical gold into investment in Gold Bonds. control over commercial banks.
 In Core Bank Solution there is no any provision
21. Exp. (b) related to non-performing assets.
Base erosion and profit shifting
24. Exp. (b)
 Base erosion and profit shifting refer to the
phenomenon where companies shift their profits Payments banks
to other tax jurisdictions, which usually have  A Payments Bank is a “differentiated bank” set-up
lower rates, thereby eroding the tax base in India. under the guidelines issued by the Reserve Bank of
 The Multilateral Convention to Implement Tax India (RBI).
Treaty Related Measures to Prevent Base Erosion  Mobile telephone companies and supermarket
and Profit Shifting: chains that are owned and controlled by residents
 The Convention is an outcome of the OECD / are eligible to be promoters of Payment Banks
G20 BEPS Project to tackle base erosion and  Payments banks; will be entitled to issue ATM or
profit shifting through tax planning strategies debit cards to their customers but cannot issue a
that exploit gaps and mismatches in tax rules to credit card.
artificially shift profits to low or no-tax locations
where there is little or no economic activity,  Payments banks cannot provide loans or lending
resulting in little or no overall corporate tax being services to the customers.
paid.
25. Exp. (b)
22. Exp. (C)
Bitcoins
Marginal Cost of Funds based Lending Rate (MCLR)  It is an electronic or digital currency that works on
 The marginal cost of funds-based lending rates a peer-to-peer basis. It is decentralized and has no
(MCLR) is the minimum interest rate that a bank central authority controlling it.
can lend at.
 Bitcoins can be sent digitally to anyone who has
 The Reserve Bank of India has brought a new a bitcoin address anywhere in the globe. One
methodology of setting the lending rate by
person could have multiple addresses for different
commercial banks under the name Marginal
purposes – personal, business and the like.
Cost of Funds based Lending Rate (MCLR). It has
modified the existing base rate system from April  A bitcoin is not printed currency but is a non-
2016 onwards. reputable record of every transaction that it has

32
www.iasscore.in ECONOMY WORKBOOK

been through. All this is part of a huge ledger  Business Correspondent model is an innovative,
called the blockchain. technology-based banking model is giving people
 Since no authority controls the generation of the in remote areas of India access to formal financial
coins or tracks them, the system itself is designed in institutions.
such a way that the network maintains a foolproof The scope of activities:
system of the record of every transaction as well  Disbursal of small value credit
as tracking issuance of the currency sent without
either side knowing the identity of the other  Recovery of principal/collection of interest
 Collection of small value deposits and
26. Exp. (c) withdrawals
Statutory Liquidity Ratio  Enables the beneficiaries to draw their subsidies
 Banks have to invest a certain percentage of their and social security benefits in their villages
deposits in specified financial securities like Central  Sale of microinsurance/ mutual fund products/
Government or State Government securities. This pension products/ other third party products and
percentage is known as SLR.
 Receipt and delivery of small value remittances/
 This money is predominantly invested in other payment instruments.
government approved securities (bonds), Gold,
which mean the banks can earn some amount as 30. Exp. (a)
‘interest’ on these investments Marginal Standing Facility (MSF)
 By reducing Statutory Liquidity Ratio more  Marginal Standing Facility (MSF) is a new scheme
liquidity remains with commercial banks so may announced by the Reserve Bank of India (RBI) in its
cut their lending rates. Monetary Policy (2011-12) and refers to the penal
rate at which banks can borrow money from the
27. Exp. (b)
central bank over and above what is available to
Basel III Accord them through the LAF window.
 Basel III is an internationally agreed set of Net Demand and Time Liabilities
measures developed by the Basel Committee on
 The Net Demand and Time Liabilities or NDTL
Banking Supervision in response to the financial
shows the difference between the sum of demand
crisis of 2007-09.
and time liabilities (deposits) of a bank (with the
 The measures aim to strengthen the regulation, public or the other bank) and the deposits in the
supervision and risk management of banks. form of assets held by the other bank.
 Like all Basel Committee standards, Basel III
31. Exp. (a)
standards are minimum requirements that apply
to internationally active banks. Codex Alimentarius Commission
 Members are committed to implementing and  The Codex Alimentarius is a collection of
applying standards in their jurisdictions within international food safety standards that have been
the time frame established by the Committee. adopted by the Codex Alimentarius Commission
(the “Codex”) in collaboration with WTO.
28. Exp. (c)  The Codex is based in Rome and funded jointly by
Interest rates and investment the FAO and the WHO.
 Lower interest rates make it cheaper to borrow.  The Commission’s main goals are to protect the
This tends to encourage spending and investment. health of consumers and ensure fair practices in the
 This leads to higher aggregate demand and international food trade. The Codex Alimentarius
economic growth. This increase in aggregate is recognized by the World Trade Organization as
demand may also cause inflationary pressures. an international reference point for the resolution
of disputes concerning food safety and consumer
 The relationship between the interest rate and
protection
investment Expenditure is illustrated by the
investment curve of the economy. The curve has 32. Exp. (d)
downward slope, indicating that a drop in interest
rate, causes the investment-spending to rise. Multilateral Investment Guarantee Agency (MIGA)
 Multilateral Investment Guarantee Agency (MIGA) is
29. Exp. (c)
a specialized agency of the United Nations, formed
Business Correspondent (Bank Saathi) in 1988, with headquarters in Washington, D.C.

33
ECONOMY WORKBOOK www.iasscore.in

International Finance Corporation (IFC) 36. Exp. (d)


 International Finance Corporation (IFC) is a  Option (d) is correct: Gold Tranche refers to the
specialized agency of United Nations (UN), amount of gold that each member country of the
affiliated with but legally separate from the International Monetary fund (IMF) contributes
International Bank for Reconstruction and as part of its membership obligations to the fund,
Development (World Bank), founded in 1956, and can readily borrow when facing economic
Headquartered in Washington, D.C. difficulties.
International Center for Settlement of Investment
Disputes (ICSID) 37. Exp. (d)

 International Center for Settlement of Investment  Non-financial debt consists of credit instruments
Disputes (ICSID) is a specialized agency of the issued by governmental entities, households and
United Nations. businesses that are not included in the financial
sector.
 A member of the World Bank Group, it was formed
in 1956 and has its headquarters in Washington,  Examples of Non-Financial Debt Debts are
D.C. contractual obligations to repay monetary
Bank for International Settlements loans, often with related interest expense. Non-
financial debt includes industrial or commercial
 Bank for International Settlements is not related
loans, Treasury bills and credit card balances.
to UNITED NATION.
They share most of the same characteristics with
 BIS is an international financial organization financial debt, except the issuers are non-financial.
owned by 60 member central banks, representing They have maturities ranging from one day to
countries from around the world including India. perpetuity, and can be used as loans to finance
It fosters international monetary and financial a company’s growth. Companies can also use
cooperation and serves as a bank for central more sophisticated debt instruments for hedging
banks. Its headquarter is in Basel, Switzerland purposes. So, statement 2 and 3 are also correct.
33. Exp. (b) 38. Exp. (b)
United Nations Framework Convention on Climate
 MSF or Marginal Standing Facility enables banks
Change (UNFCCC)
to borrow funds from RBI (Reserve Bank of India)
 The UNFCCC, signed in 1992 at the United Nations in emergency situations when their liquidity
Conference on Environment and Development absolutely dries up.
also known as the Earth Summit, the Rio Summit
 This short-term borrowing scheme facilitates the
or the Rio Conference in Rio de Janerio. scheduled banks to get funds from the central
 The UNFCCC entered into force on March 21, 1994, bank of India overnight in case of serious cash
and has been ratified by 197 countries. shortage by offering their approved government
securities.
34. Exp. (a)  MSF rate or Marginal Standing Facility rate is the
International Development Association (IDA) interest rate at which the Reserve Bank of India
provides money to the scheduled commercial
 The International Development Association (IDA) banks who are facing acute shortage of liquidity.
is the part of the World Bank that helps the world’s
 If RBI decided to adopt an expansionist monetary
poorest countries.
policy, it will not increase the MSF rate.
 It was established in 1960 under the administration
of International Bank for Reconstruction and 39. Exp. (b)
Development by lending to developing countries  A District Co-operative Central Bank (DCCB) is a
that suffer from the lowest gross national income, cooperative bank operating at the district level in
from troubled creditworthiness, or from the various parts of India to provide banking to the
rural hinterland for the agricultural sector with
lowest per capita income.
the branches primarily established in rural and
35. Exp. (a) semi-urban areas.
 Statement 1 is incorrect: SCBs and RRBs provide
 Yes, the Commonwealth has no charter, treaty or
more credit than DCCBs.
constitution.
 Statement 2 is correct: DCCB’s main function
 Mozambique and Rwanda are also its members, can be considered as money pooling for the PACS.
who were never colonies of Britain or never DCCBs act as a balancing center for Primary
become under rule of BRITISH Agricultural Credit Societies (PACS). This is done

34
www.iasscore.in ECONOMY WORKBOOK

by diverting the surplus funds of some societies to  There are a number of economic factors that
those who face shortages of funds. impact Treasury yields, such as interest rates,
inflation, and economic growth.
40. Exp. (c)
 The major factors affecting the yield are the
 Statement 1 is correct: Commercial paper, in monetary policy of the Reserve Bank of India,
the global financial market, is an unsecured especially the course of interest rates, fiscal
promissory note with a fixed maturity.
position of the government, global markets,
 Statement 2 is incorrect: Regulated by the economy and the inflation.
Reserve Bank of India, a Certificate of Deposit is a
type of money market instrument issued against  US Bond yields
the funds deposited by an investor with a bank in 44. Exp: (d)
a dematerialized form for a specific period of time.
CDs can only be issued to individuals, companies,  Black money eats up a part of the tax and, thus, the
fund houses, and such. government’s deficit increases.
 Statement 3 is correct: An interbank call money
45. Exp: (d)
market is a short-term money market which
allows for large financial institutions to borrow  Deficit financing is inherently inflationary. Since
and lend money at interbank rates. deficit financing raises aggregate expenditure and,
 Statement 4 is incorrect: A zero-coupon bond is a hence, increases aggregate demand, the danger of
debt security that does not pay interest but instead inflation looms large.
trades at a deep discount, rendering a profit at  Printing new currency notes increases the flow of
maturity, when the bond is redeemed for its full money in the economy. This leads to increase in
face value.
inflationary pressures which leads to rise of prices
41. Exp: (c) of goods and services in the country.

 Statement (1) is correct: RBI Governor and 46. Exp: (c)


Deputy Governors are appointed by the Central
Government.  The money created by the Federal Reserve is the
monetary base, also known as high-powered
 Statement (2) is incorrect: RBI ACt, 1934- The
money. Banks create money by making loans.
Central Government may from time to time
A bank loans or invests its excess reserves to
give such directions to the Bank as it may, after
consultation with the Governor of the Bank, earn more interest. A one-dollar increase in
consider necessary in the public interest. the monetary base causes the money supply to
increase by more than one dollar. The increase in
 Statement (3) is correct: RBI Act- 1934 -SGovernor
the money supply is the money multiplier.
and in his absence the Deputy Governor
nominated by him in this behalf, shall also have 47. Exp: (b)
powers of general superintendence and direction
 CDSL is promoted by BSE which later divested its
of the affairs and the business of the Bank, and
may exercise all powers and do all acts and things stakes among nationalized banks
which may be exercised or done by the Bank. 48. Exp: (b)
42. Exp: (b)  A lender of last resort (LoR) is an institution,
 The recent Banking Regulation (Amendment) usually a country’s central bank, that offers loans
Act 2020 enables the RBI to get all the powers, to banks or other eligible institutions that are
including those hitherto exclusively with the experiencing financial difficulty or are considered
registrar of cooperative societies. However, highly risky or near collapse.
powers of registrar continue to be with him but
49. Exp. (a)
the powers of RBI override those of registrar.
 Statement 1 is correct: Since these bonds
 UCBs are permitted to raise equity share capital,
preference shares and debt instruments. provide no risk of capital loss, it can offer a lesser
rate of interest (coupon) as interest is directly
 The applicability of banking laws to cooperatives proportional to risk.
societies since March 1, 1966 ushered in ‘duality
of control’ over UCBs between the Registrar  Statement 2 is correct: Inflation indexed bonds
of Cooperative Societies/Central Registrar of provide protection to investors from uncertainty
Cooperative Societies and the Reserve Bank of India. regarding inflation.
 Statement 3 is incorrect: Interest or inflation
43. Exp: (d)
compensation both are taxable. There is no special
Bonds yields in India are affected by treatment for these bonds.

35
ECONOMY WORKBOOK www.iasscore.in

50. Exp. (d)  To advise the Central Government on matters


relating to appointments, confirmation or
 Indirect transfers refer to situations where when
foreign entities own shares or assets in India, the extension of tenure and termination of services of
shares of such foreign entities are transferred the Directors of nationalised banks.
instead of a direct transfer of the underlying assets  To build a data bank containing data relating to
in India. the performance of nationalised banks and its
officers.
51. Exp. (c)
 To advise the Central Government on the
 Statement 1 is correct: A share of household formulation and enforcement of a code of
financial savings goes to the government
conduct and ethics for managerial personnel in
borrowings, as part of public accounts of India. It
nationalised banks.
mainly consists of provident funds.
 To advise the Central Government on evolving
 Statement 2 is correct: Dated securities means
suitable training and development programs for
regular government bonds, whereas T-bills are
managerial personnel in nationalised banks.
considered separately. Dated securities issued at
market related rates comprise a large share of  So By Elimination the other two statements are
internal debt. correct.

52. Exp. (a) 54. Exp. (c)


 Statement 1 is correct: Tight monetary policy of  Both the statements are correct.
the US federal Reserve could lead to capital flight. Convertible Bonds
It is also called ‘Taper Tantrum’.
 Convertible bonds are hybrid securities that offer
 Statement 2 is correct: Interest risk associated
investors the best of both stocks and bonds. Like
with capital flight: capital flight increases the
any other kind of bond, they provide a guaranteed
interest rate on firms with Existing borrowings.
income stream and pay back the amount you
 Statement 3 is incorrect: Capital Risk associated originally lent the company.
with capital flight: Devaluation of domestic
currency will increase the risk with external  Convertible bonds typically carry lower interest
commercial borrowings.It will increase the rates payments than straight corporate bonds—
liability of domestic firms in terms of domestic the savings in interest expense can be significant.
currency. Investors accept the lower interest payments
because the conversion option offers the
53. Exp. (b) opportunity to benefit from increases in the stock
 Statement 1 is incorrect: It is not mandated that price. Hence statement 1 is correct.
the Governor of RBI will be the chairman of Bank  Governments might use indexation as a way to
Board Bureau (BBB). potentially alleviate the negative effects inflation
 Statements 2 & 3 are correct: The BBB is can have on the recipients of transfer payments
empowered to select the heads of Public sector and entitlements. Social Security payments, for
Banks. It also helps PSBs to develop strategies and example, are indexed to the annual increase in the
capital raising plans. Consumer Price Index.
 The broad agenda of the Banks Board Bureau was 55. Exp. (d)
the administration of state-owned lenders. Its
functions involve:  Reserve Bank of India is responsible for
maintaining price stability and controlling
 providing assistance to Public Sector Banks to inflation.
restructure their business strategies
 It is the responsibility of MPC in India, but RBI
 Assisting banks with the strategies to deal with
itself plays a large role in MPC and also carries out
issues of bad loans or stressed assets
task of monetary policy committee.
 Strategies for raising capitals through innovative
 So RBI is the correct answer if MPC is not given in
financial instruments and methods
the options.
 Recommendations to the government on top-
level appointments like full-time Directors, non- 56. Exp. (a)
Executive Chairman in PSBs.  NFTs typically contain references to digital
 suggest plans for consolidation and merger with files such as photos, videos, and audio. Because
other banks while they are trapped in the problem NFTs are uniquely identifiable, they differ from
of high collective gross NPAs. cryptocurrencies, which are fungible. The market

36
www.iasscore.in ECONOMY WORKBOOK

value of an NFT is associated with the digital file it and real estate.
references.  “Tokenizing” these real-world tangible assets
 Non-fungible tokens (NFTs) are cryptographic makes buying, selling, and trading them more
assets on a blockchain with unique identification efficient while reducing the probability of fraud.
codes and metadata that distinguish them from  NFTs can also function to represent individuals’
each other. identities, property rights, and more.
 Statement 3 is incorrect: Unlike cryptocurrencies,  The distinct construction of each NFT has the
they cannot be traded or exchanged at potential for several use cases. For example, they
equivalency. This differs from fungible tokens are an ideal vehicle to digitally represent physical
like cryptocurrencies, which are identical to each assets like real estate and artwork. Because they
other and, therefore, can serve as a medium for are based on blockchains, NFTs can also work to
commercial transactions. remove intermediaries and connect artists with
 NFTs are unique cryptographic tokens that exist audiences or for identity management. NFTs can
on a blockchain and cannot be replicated. remove intermediaries, simplify transactions, and
 NFTs can represent real-world items like artwork create new markets.

ECONOMIC SECTORS

1. Exp. (c) Leveraging UNOPS’ mandate and its core strengths


in infrastructure, procurement and project
 Statement 1 is correct: Coal is the most important
management, S3i - Sustainable Investments
and abundant fossil fuel in India. It accounts for
in Infrastructure and Innovation is dedicated
55% of the country’s energy need. The Atomic
to helping make the SDGs a reality. Focusing on
Minerals Directorate for Exploration and Research
affordable housing, renewable energy and health
(AMD) has identified a total of 3,50,438 tonne (t)
infrastructure, the S3i sustainable infrastructure
in situ U3O8 (2,97,170t U) uranium deposits in
investments aim to crowd public and private
in forty four (44) uranium deposits in Andhra
financing into large-scale infrastructure projects
Pradesh, Telangana, Jharkhand, Meghalaya,
in regions that have struggled to attract capital. In
Rajasthan, Karnataka, Chhattisgarh, Uttar Pradesh,
addition to money, building a sustainable future
Uttarakhand, Himachal Pradesh and Maharashtra.
requires new ideas.
 Statement 2 is incorrect: Uranium enriched
 The UNOPS S3i Innovation initiative boosts
to at least 60% is not typically required for the
innovation with real impact, aiming to close the
production of electricity. The enrichment level
gap between market needs and capacities. The
necessary for electricity generation in nuclear
S3i Innovation Program helps start-ups with high
power plants is much lower. Most commercial
impact potential, innovation height and scalable
nuclear reactors use uranium fuel enriched to
solutions, and brings together people from
around 3% to 5% of the isotope uranium-235 (U-
corporations, institutions, and academia to solve
235), with the remaining uranium consisting
real problems on the ground.
mostly of the non-fissile isotope uranium-238
(U-238). This level of enrichment is sufficient for 3. Exp. (d)
sustained nuclear fission reactions that release
energy in the form of heat, which is then used to  Statement 1 is incorrect: East-West Corridors
generate electricity. runs through Porbandar–Rajkot-Samakhiali–
Radhanpur (in Gujarat)–to Bongaigaon -
2. Exp. (d) NalbariBijni–Guwahati–Nagaon–Dabaka–Silchar)
in Assam).
All four options are correct.
 Statement 2 is incorrect: India–Myanmar–
 Sustainable investing is very high on the UN agenda,
Thailand Trilateral Highway (IMT Highway, is a
as it has become evident that the Sustainable
highway under upgrade under India’s Look East
Development Goals (SDGs) cannot be achieved
policy that will connect Moreh (Manipur), India
without substantial input from the private
with Mae Sot, Thailand via Myanmar.
sector. The financing gap is particularly large
in infrastructure, where it is further hampered  Statement 3 is incorrect: BCIM corridor proposes
by a lack of bankable infrastructure projects. to link Kunming in China’s Yunnan province with

37
ECONOMY WORKBOOK www.iasscore.in

Kolkata, passing through nodes such as Mandalay to Rs. 1 Crore of investment and Rs. 5 Crore of
in Myanmar and Dhaka in Bangladesh before turnover. The limit of small unit was increased
heading to Kolkata. to Rs. 10 Crore of investment and Rs 50 Crore of
turnover. Similarly, the limit of medium unit was
4. Exp. (c)
increased to Rs. 20 Crore of investment and Rs.
 The Democratic Republic of Congo is the leader 100 Crore of turnover. The Government of India
among the world’s top cobalt-producing countries, on 01.06.2020 decided for further upward revision
accounting for more than 70% of global output. of the MSME Definition. For medium Enterprises,
now it will be Rs. 50 Crore of investment and Rs.
5. Exp. (b)
250 Crore of turnover.
 Statement 1 is correct: Kamarajar Port, located
on the Coromandel Coast about 24 km north of  Option 2 is correct: The Government has taken
Chennai Port, Chennai, it is the 12th major port measures for easing access to credit for MSMEs. As
of India, and the first port in India which is a reported by Reserve Bank of India (RBI) some of
public company. the measures taken by RBI for improving flow of
credit to MSME sector are as under:
 Statement 2 is correct: Mundra Port is the India’s
first private port and largest container port,  Priority Sector Lending Guidelines: In terms
located on the northern shores of the Gulf of Kutch of Master Direction on ‘Priority Sector Lending
near Mundra, Kutch district, Gujarat. (PSL) – Targets and Classification’ dated
September 4, 2020, all bank loans to MSMEs
 Statement 3 is incorrect: Jawaharlal Nehru conforming to the conditions prescribed therein
Port Trust (JNPT) Known as NhavaSheva, JNPT qualify for classification under priority sector
is the largest container port in India (not lending.
Vishakhapatnam) and one of the most essential
 Collateral requirements of MSME units:
subcontinents harbours on the Western coast.
Scheduled Commercial Banks have been
6. Exp. (a) mandated not to accept collateral security in the
case of loans up to ?10 lakh extended to units in
Stability and Growth Pact the MSE sector.
The Stability and Growth Pact (SGP) is a set of rules  Trade Receivables Discounting System
designed to ensure that countries in the European (TReDS): In order to address the problem of
Union pursue sound public finances and coordinate delayed payments to MSMEs, RBI has issued
their fiscal policies. guidelines for setting up and operating Trade
 The Stability and Growth Pact (SGP) is a set of fiscal Receivables Discounting System (TReDS).
rules designed to prevent countries in the EU from The scheme facilitates the financing of trade
spending beyond their means. receivables of MSMEs from corporate and other
buyers, including government departments
7. Exp. (d) and public sector undertakings (PSUs) through
multiple financiers electronically.
As per the WTO data released in 2022, India’s share
in global exports for merchandise was 1.9 % and in 9. Exp. (c)
global imports was 4.1 %. India’s share in global
 The economic cost comprises the procurement
exports was 3.5 % and imports was 3.2 %.
price of foodgrains, costs related to procurement
8. Exp. (b) (such as statutory taxes, labour costs, mandi fees,
 Option 1 is incorrect: Union Ministry of Micro, and so on) and costs of distribution (including
Small and Medium Enterprises (M/o MSMEs) freight, storage and administration).
has issued Gazette notification to pave way for 10. Exp. (d)
implementation of the upward revision in the
definition and criteria of MSMEs in the country.  India’s external debt is held in multiple currencies,
The new definition and criterion will come into the largest of which is the United States dollar. As
effect from 1st July, 2020. on 31 December 2017, 48.2% of the country’s debt
was held in U.S. dollars. The rest of the debt is held
 After 14 years since the MSME Development
in Indian rupees (37.3%), special drawing rights
Act came into existence in 2006, a revision
(5.7%), Japanese yen (4.6%), Euros (3.2%) and
in MSME definition was announced in the
other currencies (1%).
Atmnirbhar Bharat package on 13th May, 2020.
As per this announcement, the definition of Micro  Most of India’s external debt is in form of ECB and
manufacturing and services units was increased NRI deposits.

38
www.iasscore.in ECONOMY WORKBOOK

 The fourth five year plan for the first time the
S. No. Component External Percentage
debt (US$) share financial sector was included as an integral part
of the plan.
 Major reform taken during this period was
1. Multilateral 56,021 10.9%
Nationalization of Banks and abolition of privy
million
purses.
2. Bilateral 23,371 4.6%  Financial sector was included for the first time in
million fourth five year plan itself.

3. IMF loans 5,666 million 1.1% 14. Exp. (a)

4. Export credit 9,390 million 1.8% Purchasing Power Parity (PPP)

5. Commercial 196,861 38.3%  The purchasing power of a currency refers to the


borrowings million quantity of the currency needed to purchase a
given unit of a good, or common basket of goods
6. NRI deposits 123,315 24.0% and services.
million  Purchasing power is clearly determined by
the relative cost of living and inflation rates in
7. Rupee debt 1,205 million 0.2%
different countries.
Long-term 415,829 81.0%
 Purchasing power parity means equalising the
debt million
purchasing power of two currencies by taking
Short-term 97,609 19.0% into account these cost of living and inflation
debt million differences.
Total 513.4 billion 100%  PPP exchange rates are calculated by comparing
the prices of the same basket of goods and services
in different countries.
11. Exp. (b)
 In terms of PPP dollars, India is the third largest
 Currency crisis risk is reduced by increase in
economy in the world.
foreign exchange inflow in the current account
of BoP. In this question 1 and 3 are inflows in 15. Exp. (d)
the current account and hence reduce the risk,
whereas 2 are not related to currency risk directly. India’s top 15 Agricultural Export Commodities
Quantity: ‘000 tonnes Value in Rs. Crores
12. Exp. (d)
S.No. Commodity 2013-14 2014-15 2015-16
 P-Notes or Participatory Notes are Overseas Qty Value Qty Value Qty Value
Derivative Instruments that have Indian stocks 1. Marine Products 1001 30627 1073 33685 976 31183
as their underlying assets. They allow foreign 2. Buffalo Meat 1366 26458 1476 29283 1314 26682
investors to buy stocks listed on Indian exchanges 3. Rice-Basmoti 3754 29292 3702 27599 4045 22714
through FIIs without being registered. 4. Spices 897 15146 923 14842 821 16374
Rice (Other Than
13. Exp. (a) 5. 7148 17795 8226 20336 6374 15286
Basmoti)
A sizeable increase in national income so as to cotton Raw Incld.
 6. 1948 22338 1143 11643 1346 12816
Waste
raise the level of living.
7. Sugar 2478 7179 1954 5327 3826 9772
 Rapid industrialization of the country with 8. Coffee 254 4799 221 4973 256 5123
particular emphasis on the development of basic 9. Cashew 121 5095 135 5566 103 5025
and Capital Goods industries. 10. Fresh Vegetables 2292 5384 2019 4612 1872 47663
 A large expansion of employment opportunities 11. Tea 250 4873 215 4166 247 4719
12. Castro Oil 545 4364 547 4710 587 4616
by developing labour-intensive projects and small
Tobacco
scale industries. 13. 237 4783 220 4163 215 4371
Unmanufactured
 Reduction in inequalities of income and 14. Groundnut 510 3188 708 4675 537 4039
distribution. 15. Fresh Fruits 525 3646 484 3148 573 3918
 To attain the annual growth rate of 5%. Source: DGCIS, Kolkata

39
ECONOMY WORKBOOK www.iasscore.in

India’s Top 15 Agricultural Import Commodities 17. Exp. (a)

Quantity: ‘000 tonnes Value in Rs. Crores  The Indira Gandhi administration nationalized
Commodity 2013-14 2014-15 2015-16 coal mining in phases – coking coal mines in 1971–
S. No. 72 and non-coking coal mines in 1973. With the
Qty Value Qty Value Qty Value
enactment of the Coal Mines (Nationalization) Act,
Vegetable
1.
Oils
7943 44038 11548 59094 15639 68630 1973, all coal mines in India were nationalized on
1 May 1973.
2. Pulses 3178 11037 4585 17063 5798 25619
 Under the new policy, mines will be auctioned to
3. Fresh Fruits 769 7716 858 9544 836 11013 the firm offering the highest per tonne price.
4. Cashew raw 776 4668 941 6600 962 8701  Due to high demand and poor average quality,
5. Spices 156 3452 161 4392 191 5382 India is forced to import high quality coal to
6. Sugar 881 2287 1539 3668 1943 4038
meet the requirements of steel plants. India’s coal
imports have risen from 49.79 million metric tons
Alcoholic
7.
Beverages
2076 0 2508 0 2915 (0.05488 billion short tons) in 2007–08 to 190.95
million metric tons (0.21049 billion short tons) in
Cotton Raw
8. 181 2376 259 3101 232 2563 2016–17.
Incld. Waste
Mise
9. Processed 1474 1749 0 1799
18. Exp. (b)
Items  India is the largest exporter of rice in the world
Cocoa since the last decade while China is largest
10. 52 1072 65 1551 56 1399
Products
producer of rice. Below are the 15 countries that
11. Wheat 11 27 29 61 516 870 exported the highest dollar value worth of rice
12. Coffee 60 729 75 930 66 802 during 2018.
Fruits /  India: US$7.4 billion (30.1% of total rice exports)
13. Vegetables 8 449 14 611 14 702
Seeds  Thailand: $5.6 billion (22.7%)
14.
Marine
31 411 28 452 50 635  Vietnam: $2.2 billion (9%)
Products
Cereal  Pakistan: $2 billion (8.2%)
15. 53 419 63 569 61 573
Preparations  United States: $1.7 billion (6.9%)
Source: DGCIS, Kolkata
 China: $887.3 million (3.6%)

16. Exp. (b) 19. Exp. (b)

 The main objectives of the Land Reforms: Minimum Support Price

 To make redistribution of Land to make a  MSP is the minimum price set by the Government
socialistic pattern of society. Such an effort will at which farmers can expect to sell their produce
reduce the inequalities in ownership of land. for the season.

 To ensure land ceiling and take away the  When market prices fall below the announced
surplus land to be distributed among the small MSPs, procurement agencies step in to procure the
and marginal farmers. crop and ‘support’ the prices

 To legitimize tenancy with the ceiling limit.  The government announces MSP on 23 crops.
These include seven cereal crops (paddy, wheat,
 To register all the tenancy with the village jawar, bajra, maize, ragi and barley), five pulse
Panchayats.
crops (gram, tur, moong, urad and lentil), seven
 To establish relation between tenancy and oilseeds (groundnut, sunflower seed, soya bean,
ceiling. rapeseed, mustard, safflower, nigerseed and
 To remove rural poverty. seasmum), copra (dried coconut), cotton, raw jute
and sugarcane.
 Proliferating socialist development to lessen
social inequality 20. Exp. (d)
 Empowerment of women in the traditionally National Investment and Infrastructure Fund
male driven society. (NIIF)
 To increase productivity of agriculture.  National Investment and Infrastructure Fund
 To see that everyone can have a right on a piece (NIIF) is a fund created by the Government of
of land. India for enhancing infrastructure financing in
 Protection of tribal by not allowing outsiders to the country.
take their land.  National Investment and Infrastructure Fund

40
www.iasscore.in ECONOMY WORKBOOK

(NIIF) is under Department of Economic Affairs. 25. Exp. (b)


 It was set up in Budget 2015, with Rs.40k crores. Pradhan Mantri Fasal Bima Yojna (PMFBY)
21. Exp. (a)  The Pradhan Mantri Fasal Bima Yojna (PMFBY)
was introduced on 14th January 2016, in a move
Standard Mark aimed at reducing agricultural distress and
 Standard mark and certification from the Bureau farmer’s welfare without having to affect hefty
of Indian Standards (BIS) are mandatory for all hikes in the Minimum Support Prices (MSP) of
types of automotive tires and tubes. agricultural products prices due to Monsoon
AGMARK fluctuations induced risks.

 AGMARK is a certification mark employed on  Under the scheme, farmers will have to pay a
agricultural products in India, assuring that uniform premium of two percent for all Kharif
they conform to a set of standards approved by crops and 1.5 percent for all Rabi crops. For annual
the Directorate of Marketing and Inspection an commercial and horticultural crops, farmers will
have to pay a premium of 5 percent.
attached Office of the Department of Agriculture,
Cooperation and Farmers Welfare under Ministry  Post-harvest Coverage is available up to a
of Agricultural & Farmers Welfare. maximum period of 14 days from harvesting
for those crops which are kept in “cut & spread”
22. Exp. (c) condition to dry in the field after harvesting,
against specific perils of cyclone / cyclonic rains,
National Agriculture Market (NAM)
unseasonal rains throughout the country.
 National Agriculture Market (NAM) is a pan-India
electronic trading portal which networks the 26. Exp. (d)
existing APMC mandis to create a unified national Ujjwal DISCOM Assurance Yojana
market for agricultural commodities.
 The government of India launched the Ujwal
 The NAM Portal provides a single-window service DISCOM Assurance Yojana (UDAY) in 2015 for
for all APMC related information and services. operational and financial turnaround of State-
This includes commodity arrivals & prices, buy owned Power Distribution Companies (DISCOMs).
& sell trade offers, provision to respond to trade The scheme is seen as a path-breaking reform for
offers, among other services. realizing Prime Minister’s vision of affordable and
accessible 24×7 Power for All.
23. Exp. (c)
 UDAY is a comprehensive scheme providing
Quality Council of India (QCI) measures for both revenue-side efficiency and
 Quality Council of India (QCI) was set up in 1997 as cost-side efficiency.
an autonomous body.
27. Exp. (b)
 It was set up jointly by the Government of India
 The Eight Core Industries comprise nearly 40.27%
and the Indian Industry represented by the three
of the weight of items included in the Index of
premier industry associations i.e.Associated
Industrial Production (IIP). These are Electricity,
Chambers of Commerce and Industry of India
steel, refinery products, crude oil, coal, cement,
(ASSOCHAM),Confederation of Indian Industry
natural gas, and fertilizers.
(CII) and Federation of Indian Chambers of
Commerce and Industry (FICCI).  In India, there are eight core sectors comprising of

 The chairman of QCI is appointed by the Prime  Refinery products (28.04%)


Minister on the recommendation of the industry  Electricity (19.85%)
to the government.  Steel (17.92%)
24. Exp. (a)  Coal (10.33%)

 India’s first ‘National Investment and  Crude oil (8.98%)


Manufacturing Zone  Natural gas (6.88%)
 Andhra Pradesh is set to house India’s first national  Cement (5.37%)
investment and manufacturing zone after the  Fertilizers (2.63%)
state assured the Centre of availability of 10 sq km
of land in one place in Prakasham district. 28. Exp. (a)
 The National Investment & Manufacturing Zones Fair and Remunerative Price (FRP)
(NIMZs) are an important instrumentality of the  It is the minimum price at which rate sugarcane is
manufacturing policy. to be purchased by sugar mills from farmers.

41
ECONOMY WORKBOOK www.iasscore.in

 FRP is fixed by Union government on basis of projects relating to new and renewable sources of
recommendations of Commission for Agricultural energy and energy efficiency/conservation with
Costs and Prices (CACP), an attached office of the motto: “ENERGY FOR EVER”
Union Ministry of Agriculture & Farmers Welfare.
33. Exp. (c)
29. Exp. (b)
Coking coal
Agricultural Produce Market Committee Act
 The coal found in India is mainly of non-coking
 The Agricultural Produce Market Committee quality and hence coking coal has to be imported.
Act empowers state governments to notify the 70% of the steel produced today uses coal. Coking
commodities, and designate markets and market coal is a vital ingredient in the steel making
areas where the regulated trade takes place. process.
 The Act also provides for the formation of  Coking Coal is being imported by Steel Authority of
agricultural produce market committees (APMC) India Limited (SAIL) and other Steel manufacturing
that are responsible for the operation of the units mainly to bridge the gap between the
markets. requirement and indigenous availability and to
improve the quality of production.
30. Exp. (c)
Consumer Price Index Numbers for Industrial 34. Exp. (b)
Workers Sustainable Sugarcane Initiative (SSI)
 Consumer Price Index Numbers for Industrial  Sustainable Sugarcane Initiative is an innovative
Workers is being compiled and maintained by method of sugarcane production using less seeds,
Labour Bureau, Ministry of Labour & Employment less water and optimum utilization of fertilizers
in respect of All-India. and land to achieve more yields, so Seed cost is
 Consumer Price Index Numbers for Industrial very low
Workers (CPI-IW ) tries to measure the alterations  The productivity of cane under SSI can be
over a time period on the prices of a fixed basket of enhanced by practicing drip irrigation with
goods and services utilized by Industrial Workers. fertigation. Based on the soil type, drip irrigation
31. Exp. (b) can be scheduled daily or once in three days.
Fertigation can be done at ten days interval.
Accelerated Irrigation Benefits Programme
 In Sustainable Sugarcane Initiative while
 Accelerated Irrigation Benefits Programme producing sugarcane NPK (fertilizers) can be
Launched in 1996-97 to expedite the completion of applied at the rate of 112 kg, 25 kg and 48 kg per
ongoing irrigation projects. acre, respectively through both inorganic and
 Under this scheme, Centre gives loan assistance organic methods.
to the States to help them complete some of the
 Wider spacing under SSI provides scope
incomplete major/medium irrigation projects
for intercropping with vegetables, pulses,
which are at an advanced stage of completion.
watermelon, cucumber, and green manure
Command Area Development Programme crops. Intercropping facilitates weed control, soil
 The Command Area Development Programme enrichment, and additional income.
(CAD) was started initially in December 1974
to improve the irrigation potential utilization 35. Exp. (b)
and optimize the agricultural production and Special Safeguard Mechanism (SSM)
productivity through an integrated and coordinated
 Special Safeguard Mechanism (SSM) is a
approach of efficient water management
protectionist tool under WTO that will allow
32. Exp. (c) developing countries to increase tariffs
temporarily to suppress import surges or price
Indian Renewable Energy Development Agency
falls.
Limited (IREDA)
 It is especially used for agricultural imports that
 Indian Renewable Energy Development Agency
cause damages to the country’s farmers.
Limited (IREDA) is a Mini Ratna (Category–I)
Government of India Enterprise under the 36. Exp. (d)
administrative control of Ministry of New and
Renewable Energy (MNRE). Growth of manufacturing sector
 IREDA is a Public Limited Government Company  The NMP provides for promotion of clusters and
established as a Non-Banking Financial Institution aggregation, especially through the creation of
in 1987 engaged in promoting, developing and National Investment and Manufacturing Zones
extending financial assistance for setting up (NIMZ).

42
www.iasscore.in ECONOMY WORKBOOK

 The criterion adopted for determining the ease  The scheme is demand-driven and would facilitate
of doing business is the efficacy of these reforms food processing units to meet environmental,
through Single window clearance. This system was safety and social standards
introduced under the “Make in India” initiative in
2014.It involves routing of all approvals required 40. Exp. (c)
by an enterprise to set up a business through a Statutory minimum price (SMP)
hassle-free common application window.
 The statutory minimum price (SMP) is announced
 The Technology Acquisition and Development by the central government based on the cost
Fund (TADF) Scheme is aimed at facilitating the of cultivation estimated by the Commission for
acquisition of clean, green and energy-efficient Agricultural Costs and Prices (CACP). This is
technologies by Micro Small and Medium the basic price which the sugar mills must pay
Enterprises. sugarcane growers.

37. Exp. (c) Essential Commodities Act (ECA)

Eight Core Industries  The Essential Commodities Act (ECA) was enacted
by the Central Government in 1955 to control and
 It contains index, production, and growth of Eight regulate trade and prices of commodities declared
Core Industries. essential under the Act.
 Eight Core Industries are Electricity, steel, refinery  Seven major commodities are covered under the
products, crude oil, coal, cement, natural gas, and act:
fertilizers.
 Food stuff including edible oil and seeds, vanaspati,
38. Exp. (b) pulses, sugarcane and its products like, khandsari
and sugar, rice paddy
Coal import
41. Exp. (a)
 The issue for India is that the gap between domestic
supply and demand is growing at a phenomenal Limited Liability Partnership firm
rate. This gap is being filled by imported coal.
 Every LLP shall have at least 2 partners and there
 Coal imports have increased largely because is no limit for the maximum numbers of partners.
of demand from new power plants which are The partners to LLP can be an individual as well as
designed to use only high grade imported coal. body corporate
 Low quality of Indian coal (with high ash content of  LLP is a blend of body corporate and a separate
30-40%), the inability of Coal India Ltd to increase legal entity from its partners which has a perpetual
production are key reasons for surge in imports. succession.
 Further, transport issues and poor development of  The partners of LLP have the right to manage
technologies to increase the calorific value of local the business directly, unlike the corporate
coal also make imported coal attractive. shareholders which thereby means no Owner/
manager distinction.
 Also, Steel companies need a large quantity of
 One partner is not responsible or liable for another
coking coal which has to be imported.
partner’s, misconduct or negligence;
39. Exp. (d)  LLP should be formed with a profit motive. The
Mega food parks rights and duties of partners in an LLP are to be
governed by the agreement between partners.
 The Ministry of Food Processing Industries in
India is promoting the Mega Food Park Scheme 42. Exp. (a)
under the Pradhan Mantri Kisan SAMPADA
Special Economic Zone (SEZ)
Yojana, a comprehensive package that will result
in creation of modern infrastructure with efficient  Special Economic Zone (SEZ) is a specifically
supply chain management from farm gate to retail delineated duty free enclave and shall be deemed
outlet. to be foreign territory for the purposes of trade
operations and duties and tariffs in India.
 It will not only provide a big boost to the growth of
food processing sector in the country but also help  The Special Economic Zones Act, 2005, provides
in providing better returns to farmers and is a big the legal framework for establishment of Special
step towards doubling of farmers income, creating Economic Zones and also for units operating in
huge employment opportunities especially in the such zones.
rural areas, reducing wastage of agricultural
Main objectives of establishing SEZs are:
produce, increasing the processing level and
enhancing the export of the processed foods.  Generation of additional economic activity

43
ECONOMY WORKBOOK www.iasscore.in

 Promotion of exports of both goods and services distort trade. It states that WTO members may
not apply any measure that discriminates against
 Generation of employment opportunities
foreign products or that leads to quantitative
 Promotion of investment from foreign as well as restrictions, both of which violate basic WTO
domestic players principles.
 Development of infrastructure facilities  Statement 2 is incorrect: TRIMS applies only to
measures that affect trade in goods.
43. Exp. (c)
 Statement 3 is correct: The Agreement is
Statement 1 is incorrect: India’s merchandise
not concerned with the regulation of foreign
exports in August stood at $22.70 billion, whereas,
investment.
India’s merchandise exports stood at US$ 19.05 billion.
Statement 2 is incorrect: The following product 46. Exp. (c)
groups represent the highest dollar value in India’s  Public expenditures on agriculture include short-
import purchases during 2019. Also shown is the
term costs as well as long-term investments.
percentage share each product category represents in
Investment in agriculture and forestry includes
terms of overall imports into India.
government expenditures directed to agricultural
 Mineral fuels including oil: US$153.5 billion (32% infrastructure, research and development and
of total imports) education and training.
 Gems, precious metals: $60 billion (12.5%)
 MSP is a part of India’s Agriculture Price Policy
 Electrical machinery, equipment: $50.4 billion
 PACS are the banks situated in rural area and
(10.5%)
plays an important role in rural credit system
 Machinery including computers: $44.1 billion by performing their activities on co-operative
(9.2%) principles and also these banks are worked under
 Organic chemicals: $20.5 billion (4.3%) the District Credit Co-operative Banks.
 Plastics, plastic articles: $14.6 billion (3.1%) 47. Exp. (b)
 Iron, steel: $11.6 billion (2.4%)
 Statement 2 is correct: Bangladesh has been
 Animal/vegetable fats, oils, waxes: $9.6 billion (2%) a major textile trading partner for India, with
 Optical, technical, medical apparatus: $9.5 billion a share of more than 5% in exports and over
(2%) 7% in imports. While annual textile exports to
 Fertilizers: $7.3 billion (1.5%) Bangladesh averages US$2,000 million, imports
are worth US$400. The major items of exports are
Statement 3 is correct: India’s services exports
have increased from US$ 157.20 billion in 2014 to fibre and yarn of cotton, man-made staple fibres
US$ 205.11 billion in 2018. India’s imports of services and man-made filaments while major import
have increased from US$ 128.36 billion in 2014 to US$ items include apparel and clothing, fabric and
176.58 billion in 2018. other made up textile articles.
Statement 4 is incorrect: India posted a USD 19.8  Statement 3 is incorrect: India’s trade with
billion surplus in April-June 2020, or 3.9 percent of countries in South Asia has remained less than 4%
GDP, compared to a USD 15.0 billion deficit in the same of its global trade since the late 1980s. Bangladesh
period last year. That was the largest current account is India’s largest trading partner in South Asia,
surplus since records began in 1949 due to a sharp followed by Nepal, Sri. Lanka, Pakistan, Bhutan,
contraction in the trade deficit to USD 10.0 billion Afghanistan and the Maldives.
from USD 46.8 billion last year, as imports fell more
than exports amid the coronavirus crisis. 48. Exp. (d)

44. Exp. (a)  All Statements are correct: Steel slag contains
significant amounts of free iron, which gives the
 West Texas Intermediate (WTI) can refer to a material high density and hardness and makes it
grade or a mix of crude oil, and/or the spot price, as a suitable artificial source of aggregates for road
the futures price, or the assessed price for that construction. Slag application favors the increase
oil; colloquially WTI usually refers to the price of of pH and the availability of nutrients such as
the New York Mercantile Exchange (NYMEX) WTI Ca, Mg, and Si in the soil. Steel-slag can be used
Crude Oil futures contract or the contract itself. to produce energy-saving cement by co-grinding
with OPC clinker and blastfurnace slag.
45. Exp. (c)
49. Exp. (a)
 Statement 1 is correct: The Agreement on Trade-
Related Investment Measures (TRIMS) recognizes  Statement 1 is correct: Black Gram can be
cultivated as both kharif and Rabi crop. During
that certain investment measures can restrict and

44
www.iasscore.in ECONOMY WORKBOOK

kharif, it is cultivated throughout the country. It is 56. Exp. (a)


best suited to rice fallows during rabi in southern
 Statement 1 is correct: Acquiring new technology
and south-eastern parts of India.
is considered as capital expenditure as it will
 Statement 2 is incorrect: Gram is the most generate profit in the future and helps in creation
dominant pulse having a share of around 40 per of new assets.
cent in the total production followed by Tur/Arhar
at 15 to 20 per cent and Urad/Black Matpe and  Statement 2 is incorrect: Debt Financing and
Moong at around 8-10 per cent each. equity financing are considered under capital
expenditure.
50. Exp: (d)
57. Exp. (d)
All statements are correct
 India is the member of all the Organizations.
51. Exp: (c)  The SCO currently comprises eight Member States
(China, India, Kazakhstan, Kyrgyzstan, Russia,
 Statement 1 is correct: Fertigation can control
Pakistan, Tajikistan and Uzbekistan).
alkalinity of irrigation water
 Missile Technology Control Regime: On 27 June,
 Statement 2 is incorrect: Efficient application of
India became a full member of the MTCR after a
all phosphatic fertilisers is not possible “deal” was struck with Italy. This marks the first
 Statement 3 is correct: Increased availability of entry into any multilateral export control regime
nutrients to plants is possible for India.
 Statement 4 is correct: Reduction in leaching is  Asian Infrastructure Investment Bank: On 24
possible October 2014, twenty-one countries signed a
Memorandum of Understanding (MOU) regarding
52. Exp: (a) the AIIB in Beijing, China: Bangladesh, Brunei,
Cambodia, India, Kazakhstan, Kuwait, Laos,
 FII - Foreign investment in Indian securities has
Malaysia, Myanmar, Mongolia, Nepal, Oman,
also been made possible through the purchase
Pakistan, Philippines, Qatar, Singapore, Sri Lanka,
of Global Depository Receipts, Foreign Currency Thailand, Uzbekistan and Vietnam.
Convertible Bonds and Foreign Currency Bonds
issued by Indian issuers which are listed, traded 58. Exp. (c)
and settled overseas.  Statement 1 is correct: According to a forecast
by Price water house Coopers in February 2017,
53. Exp: (a)
Vietnam may be the fastest-growing of the world’s
 A key effect of devaluation is that it makes economies, with a potential annual GDP growth
the domestic currency cheaper relative to rate of about 5.1%, which would make its economy
other currencies. There are two implications the 10th-largest in the world by 2050.
of a devaluation. First, devaluation makes the  Statement 2 is incorrect: Vietnam is a Marxist–
country’s exports relatively less expensive for Leninist one-party state based on democratic
foreigners. Second, the devaluation makes foreign centralism.
products relatively more expensive for domestic  Statement 3 is correct: Vietnam’s economic
consumers, thus discouraging imports. This may growth is linked to its integration with global
help to increase the country’s exports and decrease supply chains and focus on exports.
imports, and may therefore help to reduce the
 Statement 4 is correct: Vietnam has low labour
current account deficit. costs and stable exchange rates which attract
global investments.
54. Exp. (b)
 Statement 1 is correct: E- commerce firms can 59. Exp. (c)
sell their own products in addition to offering  Statement (c) is correct: Labour Bureau has been
their platforms as market place. bringing out Statistics on ?Industrial Disputes,
 Statement 2 is correct: Big sellers have the limit Closures, Retrenchments and Lay-offs in India?
based on the voluntary returns received every
of 25% for sale on e-commerce platform.
month from the Labour Departments of the States
55. Exp. (c) and Union Territories and the Regional Labour
Commissioners.
 Statement 1 is correct: Initiative endorsed by G20
together with Paris Club. 60. Exp. (a)
 Statement 2 is correct: It is an initiative to support  Statement 1 is correct: Coal Controller has
low income countries with unsustainable debt. been made the statistical authority with respect

45
ECONOMY WORKBOOK www.iasscore.in

to coal and lignite statistics. Entrusted with the the task of monitoring captive mines.
responsibility of carrying out Annual Coal &  Statement 3 is correct: Coal Controller is the
Lignite survey and publishing of Provisional Coal competent authority under this act to hear any
Statistics and Coal Directory of India. objection to the Central Government’s Notification
 Statement 2 is correct: Functioning of the Coal relating to acquisition of coal bearing land and to
Controller’s Organisation (CCO)is entrusted with furnish his reports to Central Govt.

SOCIO-ECONOMIC SECTORS

1. Exp. (b) arrangements for specific population groups like


government employees and factory workers. Private
 Option (b) is correct: “Small Farmers Large
voluntary insurance is available, but uptake is limited.
Field (SFLF)” is a collective action model to
overcome the disadvantages faced by millions of 4. Exp. (b)
small and marginal farmers due to diseconomies
of scale and lack of bargaining power in the supply Option 1 is incorrect: In India, banks are the
chain. predominant agency for delivery of micro-credit.

 This model is participatory and flexible and  In 1970, Ilaben Bhat, founder member of ‘SEWA’
allows small farmers to benefit from achieving (Self Employed Women’s Association) in
economies of scale by organizing themselves Ahmadabad, had developed a concept of ‘women
into groups and synchronizing and harmonizing and micro-finance’.
selected operations.  ‘The Annapurna MahilaMandal’ in Maharashtra
and ‘Working Women’s Forum’ in Tamil nadu and
2. Exp. (b)
many National Bank for Agriculture and Rural
 Development (NABARD)-sponsored groups
Statement 1 and 4 are correct: Bond market and have followed the path laid down by ‘SEWA’.
stock markets are part of capital market.  ‘SEWA’ is a trade union of poor, self-employed
 Statement 2 and 3 are incorrect: Call Money women workers. Since 1987 ‘Mysore Resettlement
market is concerned with short term financial and Development Agency’ (MYRADA) has
assets and Treasury bills are issued when the promoted Credit Management Groups (CMGs).
government needs money for a short period. CMGs are similar to self-help groups.
 The basic features of this concept promoted
3. Exp. (b)
by MYRADA are: 1] Affinity, 2] Voluntarism, 3]
Option 1 is correct: The primary objective of India’s Homogeneity and 4] Membership should be
health care system is to improve health status through limited to15-20 persons.
concerted policy action in all sectors and expand
 Aim of the CMG is to bestow social empowerment
preventive, promotive, curative, palliative and
to women. In 1991-92 NABARD started promoting
rehabilitative services provided through the public
self-help groups on a large scale. And it was the
health sector with focus on quality.
real take-off point for the ‘SHG movement’.
Link: https://main.mohfw.gov.in/sites/default/
 In 1993, the Reserve Bank of India also allowed
files/9147562941489753121.pdf
SHGs to open saving accounts in banks. Facility of
Option 2 is correct: All Indian citizens can get free availing bank services was a major boost to the
outpatient and inpatient care at government facilities. movement.
Under India’s decentralized approach to health care
Option 2 is correct: Self Help Groups (SHGs) are small
delivery, the states are primarily responsible for
groups of poor people. The members of an SHG face
organizing health services. Because of severe shortages
similar problems. They help each other, to solve their
of staff and supplies at government facilities, many
problems. SHGs promote small savings among their
households seek care from private providers and pay
members. The savings are kept with the bank. This is
out-of-pocket. For low-income people, the government
the common fund in the name of the SHG. The SHG
recently launched the tax-financed National Health
gives small loans to its members from its common
Protection Scheme (Ayushman Bharat-PradhanMantri
fund.
Jan ArogyaYojana, or PM-JAY), which allows them to
also get cashless secondary and tertiary care at private Option 3 is correct: The Indian micro finance sector
facilities. There are also a handful of health insurance has seen tremendous growth in the last few years. GOI

46
www.iasscore.in ECONOMY WORKBOOK

has taken initiatives to widen the reach of RRBs all 7. Exp. (b)
over India, especially in rural areas where commercial
National Food Security Act, 2013
banks and other financial institution are beyond the
reach of rural poor. Micro financing is one of the  In 2013, NFSA was enacted to provide food security
distinctive functional areas of RRBs. to 67% of the population in the form of highly
subsidized food grains at Rs. 2 and 3 per Kg for
The two important models of microfinance involving wheat and rice respectively.
credit linkages with banks in India are
 Beneficiaries are identified using SECC
 SHG - Bank Linkage Model: This model involves methodology so even non-BPL will also benefit.
the SHGs financed directly by the banks viz.,
CBs (Public Sector and Private Sector), RRBs and  The oldest woman of the family (who must be at
Cooperative Banks. least 18 years of age) will be considered the head of
the household when ration cards are being issued.
 MFI - Bank Linkage Model: This model covers
 Pregnant women and lactating mothers are
financing of Micro Finance Institutions (MFIs)
entitled to a nutritious “take-home ration” of 600
by banking agencies for on-lending to SHGs and
Calories (NOT 1600) and a maternity benefit of at
other small borrowers.
least Rs 6,000 for six months, says the official Act
5. Exp. (d)
8. Exp. (c)
Social Capital
Pradhan Mantri Kaushal Vikas Yojana (PMKVY)
 Social capital broadly refers to those factors of
effectively functioning social groups that include  Launched in 2015, it is a flagship program
such things as interpersonal relationships, a of the Ministry of Skill Development and
shared sense of identity, a shared understanding, Entrepreneurship (MSDE). National Skill
shared norms, shared values, trust, cooperation Development Corporation (NSDC) implements it
and reciprocity. with the help of training partners.

 The term generally refers to (a) resources, and  Recognition of Prior Learning (RPL): An
the value of these resources, both tangible (public individual with a certain set of skills in soft skills,
spaces, private property) and intangible (“actors”, entrepreneurship, financial and digital literacy.
“human capital”, and people), (b) the relationships with prior learning experience is assessed and
among these resources, and (c) the impact that certified under RPL with grade according to the
these relationships have on the resources involved NSQF.
in each relationship, and on larger groups. It is  Training as per National Skills Qualification
generally seen as a form of capital that produces Framework (NSQF) is provided to those who are
public goods for a common good. either school/college dropouts or unemployed
 Social capital has been used to explain the improved to align the competencies of the unregulated
performance of diverse groups, the growth of workforce of the country.
entrepreneurial firms, superior managerial 9. Exp. (b)
performance, enhanced supply chain relations,
the value derived from strategic alliances, and the Soil Health Card (SHC)
evolution of communities.  Soil Health Card (SHC) is a Government of
 Hence, the correct answer to this question is (d) India’s scheme promoted by the Department of
‘The level of mutual trust and harmony in the Agriculture & Co-operation under the Ministry of
society’. Agriculture and Farmers’ Welfare.
 It is being implemented through the Department
6. Exp. (b) of Agriculture of all the State and Union Territory
Poverty Line Governments.
 There are many challenges in marking a poverty  In Soi Health card there is no provision of loans to
line, such as determining components of poverty be granted to farmers on the basis of soil quality
line basket.  The SHC will carry crop-wise recommendations of
 There are price differentials (of constituents of nutrients and fertilizers required for the individual
basket) which vary from state to state and period farms to help farmers to improve productivity
to period. So, in a given year in India, official through the judicious use of fertilizers.
poverty lines are higher in some States than in
10. Exp. (c)
others because price levels vary from State to State
 Further, consumption patterns, nutritional needs National Pension System (NPS)
and prices of components keep on changing as per  A citizen of India, whether resident or non-resident
dynamics of macro economy and demography. can join NPS.

47
ECONOMY WORKBOOK www.iasscore.in

 Subscribers should be between 18 - 65 years of age institutions mentioned above or can apply online
as on the date of submission of his / her application. through this portal www .udyamimitra.in
 For the State Government employee’s contribution 14. Exp. (d)
through their nodal office to National Pension
System (NPS) is mandatory. Every month 10% SWAYAM
of his/ her salary (basic + DA) and equivalent  SWAYAM (Study Webs of Active–Learning for
government’s contribution will be invested in NPS. Young Aspiring Minds) is a program initiated by
 NPS is applicable to all new employees of Central Government of India and designed to achieve the
Government service except Armed Forces and three cardinal principles of Education Policy viz.,
Central Autonomous Bodies joining Government access, equity, and quality.

service on or after 1st January 2004.  The objective of this effort is to take the best
teaching-learning resources to all, including the
11. Exp. (a) most disadvantaged.
Recognition of Prior Learning Scheme  SWAYAM seeks to bridge the digital divide for
students who have hitherto remained untouched
 The Recognition of Prior Learning (RPL) by the digital revolution and have not been able to
component of PMKVY is primarily focussed on
assessing and certifying the skills of informal join the mainstream of the knowledge economy.
sector workers. Evaluation of trade learned skills 15. Exp: (a)
and certification through assessment helps the
trainees through increased mobility options to the Stand-up India
formal sector employment.  The Stand-up India scheme aims at promoting
 In certain cases, it has been observed that RPL entrepreneurship among women and scheduled
certification has helped workers negotiate better castes and tribes.
wages and open possibilities for some vertical  The scheme is anchored by Department of
progression in their careers. In almost all cases, Financial Services (DFS), Ministry of Finance,
trainees have displayed enhanced self-confidence Government of India.
and pride through attainment of skill certificates.  Small Industries Development Bank of India
(SIDBI) facilitates bank loans between 10 lakh
12. Exp. (c)
and 1 crore to at least one scheduled caste (SC)
Atal Pension Yojana or Scheduled Tribe, borrower and at least one
 The APY will focus on all citizens in the woman per bank branch for setting up a greenfield
unorganized sector, who join the National Pension enterprise.
System (NPS) administered by the Pension Fund 16. Exp: (c)
Regulatory and Development Authority (PFRDA)
and who are not members of any statutory social Pradhan Mantri Jan-Dhan Yojna (PMJDY)
security scheme for minimum guaranteed pension.  Pradhan Mantri Jan Dhan Yojana is a National
 A person can have both NPS and APY account. Mission on Financial Inclusion which has an
Further, it can be opened irrespective of the integrated approach to bring about comprehensive
spouse being mandatorily covered under NPS or financial inclusion and provide banking services
not. to all households in the country.

 There is no exit to the scheme before the age of 60.  The scheme ensures access to a range of financial
In case of death of subscriber, the spouse of the services like availability of basic savings bank
account, access to need-based credit, remittances
subscriber shall be entitled for the same amount
facility, insurance, and pension.
of pension till his or her death.
17. Exp: (c)
13. Exp. (a)
Disguised unemployment
Pradhan Mantri Mudra Yojana (PMMY)
 Disguised unemployment is a situation especially
 Pradhan Mantri MUDRA Yojana (PMMY) is a prevalent in poor and developing countries.
scheme launched on April 8, 2015 for providing
 Disguised unemployment is when too many people
loans up to 10 lakh to the non-corporate, non-farm
are employed than what is required to produce
small/micro enterprises.
efficiently. This kind of employment is not at all
 These loans are classified as MUDRA loans under productive.
PMMY. These loans are given by Commercial
 The key point to remember is that the marginal
Banks, RRBs, Small Finance Banks, MFIs and productivity of labourers under disguised
NBFCs. unemployment is zero. The labourers are
 The borrower can approach any of the lending employed physically, but not economically.

48
www.iasscore.in ECONOMY WORKBOOK

18. Exp: (c) World Bank, the Mission aims at creating efficient
and effective institutional platforms of the rural
Inclusive Governance
poor, enabling them to increase household income
 Inclusive Governance covers social and economic through sustainable livelihood enhancements and
inclusion with good governance. improved access to financial services.
Parts of Inclusive Governance  NRLM set out with an agenda to cover 7 Crore
 Non-Banking Financial Companies (NBFCs) rural poor households, across 600 districts, 6000
have played an irreplaceable role in fuelling the blocks, 2.5 lakh Gram Panchayats and 6 lakh
growth of the Indian economy and have made villages in the country through self-managed Self
a noteworthy contribution towards supporting Help Groups (SHGs) and federated institutions
the government’s agenda of extending financial and support them for livelihoods collectives in a
inclusion period of 8-10 years.
 District Planning Committee to consolidate
21. Exp. (a)
the plans prepared by the Panchayats and the
Municipalities in the district and to prepare a draft Multidimensional Poverty Index (MPI)
development plan for the district as a whole for  The Multidimensional Poverty Index was launched
social inclusion. by the UNDP and the Oxford Poverty & Human
 By Increasing, government spending on public Development Initiative (OPHI) in 2010.
health by number of government schemes make  MPI uses three dimensions and ten indicators
citizens more social inclusion. which are:
 Mid-day Meal Scheme programme supplies free  Education: Years of schooling and child
lunches on working days for children in primary enrollment (1/6 weightage each, total 2/6);
and upper primary classes in government,
government-aided, local body, Education  Health: Child mortality and nutrition (1/6
Guarantee Scheme, and alternate innovative weightage each, total 2/6);
education centers, Madarsa and Maqtabs.  Standard of living: Electricity, flooring,
drinking water, sanitation, cooking fuel and
19. Exp: (a) assets (1/18 weightage each, total 2/6)
Accredited Social Health Activist (ASHA)
22. Exp. (a)
 One of the key components of the National Rural
Health Mission is to provide every village in the Janani Suraksha Yojana (JSY)
country with a trained female community health  Janani SurakshaYojana (JSY) is a safe motherhood
activist ASHA or Accredited Social Health Activist. intervention under the National Health Mission.
 ASHAs are not trained to provide comprehensive It is being implemented with the objective of
reproductive (delivery of baby), maternal and reducing maternal and neonatal mortality by
new-born care. promoting institutional delivery among poor
pregnant women.
 They have several important responsibilities
within their communities including:  The scheme, launched on 12 April 2005 by the
Hon’ble Prime Minister, is under implementation
 Identifying and registering new pregnancies,
in all states and Union Territories (UTs), with a
births, and deaths
special focus on Low Performing States (LPS).
 Mobilizing, counseling and supporting the
 JSY is a centrally sponsored scheme, which
community to demand and seek health services
integrates cash assistance with delivery and post-
 Identifying, managing or referring cases of illness delivery care. The Yojana has identified Accredited
 Supporting health service delivery through home Social Health Activist (ASHA) as an effective link
visits, first-aid and immunizations sessions between the government and pregnant women.
 Maintaining data and participating in community- 23. Exp. (d)
level health planning Employees’ State Insurance Scheme
20. Exp: (b)  The Employees’ State Insurance Scheme is an
integrated measure of Social Insurance embodied
National Rural Livelihoods Mission (NRLM)
in the Employees’ State Insurance Act and it is
 Aajeevika - National Rural Livelihoods Mission designed to accomplish the task of protecting
(NRLM) was launched by the Ministry of Rural ‘employees’ as defined in the Employees’ State
Development (MoRD), Government of India in Insurance Act, 1948 against the impact of
June 2011. incidences of sickness, maternity, disablement and
 Aided in part through investment support by the death due to employment injury and to provide

49
ECONOMY WORKBOOK www.iasscore.in

medical care to insured persons and their families.  It excludes the capital markets which is left to
 The ESI Scheme applies to factories and other operate by itself in the open market.
establishments viz. Road Transport, Hotels,  Eleventh Five Year Plan:
Restaurants, Cinemas, Newspaper, Shops, and  Its duration was from 2007 to 2012
Educational/Medical Institutions wherein 10 or
more persons are employed  It was prepared by the C. Rangarajan
 Its main theme was “faster and more inclusive
24. Exp: (b) growth”
District Rural Development Agency (DRDA)  Its growth rate target was 8.1% but it achieved
 The District Rural Development Agency (DRDA) only 7.9%
has traditionally been the principal organ at the
District level to oversee the implementation of 27. Exp: (b)
different poverty alleviation programs. National Food Security Mission (NFSM)
 The DRDAs are expected to coordinate effectively  National Food Security Mission (NFSM) is a Central
with the Panchayati Raj Institution. Under no Scheme of GOI launched in 2007 for 5 years to
circumstances will they perform functions of PRIs. increase production and productivity of wheat,
 DRDAs must themselves be more professional and rice, pulses & oilseeds on a sustainable basis so as
should be able to interact effectively with various to ensure food security of the country.
other agencies. They are expected to coordinate
with the line department, the Panchayati Raj 28. Exp: (d)
Institutions, the banks, and other financial  Option 1 is correct: Minimum Support Price
institutions, resources required for poverty impacts the price of rice in India. The higher MSP
reduction effort in the district. would also dent Indian exporters’ competitiveness
 DRDA’s shall be their endeavor and objective in the global markets, which have turned to be a
to secure inter-sectoral and inter-departmental buyers’ market.
coordination and cooperation for reducing  Option 2 is correct: The government typically
poverty in the district. buys more than a third of the country’s rice output
 DRDAs shall ensure financial discipline in respect at a fixed price, which also has a direct impact on
of the funds received by them, whether from prices paid by traders. With this price rise, the
Central of State Governments. exports will become expensive.
 They shall also ensure that the accounts are Option 3 is correct:
properly maintained including in respect of the  Option 4 is correct: The impact of the consumer
funds allocated to banks or implementing agencies subsidy is to lower prices for consumers but to
in accordance with the guidelines of different increase the price received by producers.
programs.
29. Exp: (b)
25. Exp: (d)
 Purchase of combine harvesters, tractors and mini
 Mahatma Gandhi National Rural Employment trucks and Consumption requirements of farm
Guarantee Act households both are long term Capital Support
 The Mahatma Gandhi National Rural Employment
30. Exp: (d)
Guarantee Act, earlier known as the National
Rural Employment Guarantee Act was passed 31. Exp: (d)
on 7th September 2005 to augment employment
generation and social security in India.  Statement (1) is correct: Holding that an employer
cannot differentiate between contractual and
The objective of the Act
permanent employees, the Supreme Court has
 The objective of the Act is to enhance livelihood ruled that casual workers are also entitled to social
security in rural areas by providing at least security benefits under the Employees’ Provident
100 days of guaranteed wage employment in a Funds and Miscellaneous Provisions Act.
financial year to every household whose adult
members volunteer to do unskilled manual work. 32. Exp. (a)
 The real sector of the economy deals with the
26. Exp: (c)
production side, while the nominal economy deals
Inclusive growth with the financial side. A financial activity majorly
 Inclusive growth is ‘broad-based growth’, ‘shared support real (production) activity, but does not
growth’, and ‘pro-poor growth’. contribute itself too much except the factor income
it generates.

50

You might also like